American Government CLEP - ONLINE PRACTICE TEST 1-3

Ace your homework & exams now with Quizwiz!

Hazelwood v. Kuhlmeier (1988)

Censorship, Student Press Rights, Freedom of the Press Hazelwood East High School Principal Robert Reynolds procedurally reviewed the Spectrum, the school's student-written newspaper, before publication. In May 1983, he decided to have certain pages pulled because of the sensitive content in two of the articles, and acted quickly to remove them in order to meet the paper's publication deadline. The journalism students felt that this censorship was a direct violation of their First Amendment rights. The Supreme Court decided that Principal Reynolds had the right to such editorial decisions, as he had "legitimate pedagogical concerns."

Name of Case: Schenck v. United States Year: 1919

Constitutional Principle: - Civil liberties Why Decision is Important: - Established limits on free speech holding that this right is not absolute. - Set the 'clear and present danger' standard for when free speech can be restricted.

Name of Case: McCulloch v. Maryland Year: 1819

Constitutional Principle: - Federalism - National power / National Supremacy - The judiciary Why Decision is Important: - Supported the use of the elastic clause to expand federal power. - Established the principle of national supremacy- that the Constitution and federal laws overrule state laws when the two conflict.

All of the following departments are headed by a secretary EXCEPT a. Justice. b. State. c. Interior. d. Transportation. e. Energy.

The correct answer is A. The Justice Department is headed by the Attorney General. The Attorney General is responsible for enforcing the law and defending the interests of the United States.

Which of the following statements about Miranda v. Arizona is correct? a. It declared segregation by race unconstitutional. b. It granted women the same voting rights as men. c. It declared the recitation of the Pledge of the Union unconstitutional. d. It required that citizens be informed of their rights when being placed under arrest. e. It declared that a woman's reproductive rights were included in the conception of the right to privacy.

The correct answer is D. The Miranda decision required that arresting officers read a statement describing the defendant's due process rights. These rights include the right to an attorney and the right to remain silent.

The system of checks and balances incorporates the idea that a. federal expenditures should be measured by their effectiveness. b. legislative and executive power should pass through the judiciary. c. weakness in government is better than strength. d. leaders must earn the trust of the people. e. power must be used to offset power.

The correct answer is E. The system of checks and balances was designed to prevent abuses of concentrated power. The founders believed that power was best countered with power. The system they created balances and divides powers to produce a stable and democratic government that has thus far successfully avoided the dangers of tyranny.

Name of Case: Muller v. Oregon Year: 1908

Constitutional Principle: - Civil liberties - Federalism - Rights of women Why Decision is Important: - Let stand an Oregon law that limited women to a 10-hour work day in laundries or factories in order to protect women's health. - Stated that the need of the state to protect women's health outweighed the liberty to make a contract (a liberty that was upheld in Lochner).

Name of Case: Planned Parenthood of Southeastern Pennsylvania et al. V. Casey Year: 1992

Constitutional Principle: - Civil liberties - Rights of women Why Decision is Important: - Struck down the portions of a Pennsylvania law requiring (1) that a woman seeking an abortion must wait 24 hours between being informed about the procedure and having it performed and (2) that a married woman must inform her husband that she planned to have an abortion. - Upheld the portion of the law requiring minors to inform their parents before having an abortion.

Name of Case: Cruzan v. Director, Missouri Department of Health Year: 1990

Constitutional Principle: - Civil liberties Why Decision is Important: - 'Clear and convincing' evidence was presented to demonstrate that a Missouri woman (Cruzan) in a coma from 1983 car accident should have the right to die. Intravenous feeding was ended with court approval, and Cruzan subsequently died.

Name of Case: New York Times Co. v. United States Year: 1971

Constitutional Principle: - Civil liberties Why Decision is Important: - Gave the media more power against governmental secrecy.

Name of Case: Engel v. Vitale Year: 1962

Constitutional Principle: - Civil liberties Why Decision is Important: - Reinforced the separation of church and state. - Ruled that use of the public schools to encourage prayer or other religious practices is a direct violation of the establishment clause.

Name of Case: Watkins v. United States Year: 1957

Constitutional Principle: - Civil liberties Why Decision is Important: - Ruled that the House Un-American Activities Committee (HUAC) could not punish at will those witness who refused to cooperate.

Name of Case: Gideon v. Wainwright Year: 1963

Constitutional Principle: - Civil liberties / Due Process - Right to Counsel Why Decision is Important: - Ruled that to deny legal representation to defendants who can not afford to pay for it is a violation of those individual's constitutional rights.

Name of Case: Miranda v. Arizona Year: 1966

Constitutional Principle: - Criminal procedures - Civil liberties - Self-Incrimination Why decision is Important: - Established the requirement to inform people accused of crimes that they have the right to remain silent and receive legal representation before they say anything that can be held against them in court.

Name of Case: Heart of Atlanta Motel v. United States Year: 1964

Constitutional Principle: - Equality - National power Why Decision is Important: - Found racial segregation of private facilities engaged in interstate commerce unconstitutional.

Name of Case: Civil Rights Cases Year: 1883

Constitutional Principle: - Equality - National power. Why decision is Important: - Judged that racial discrimination by private persons did not place the "badge of slavery" of African Americans nor keep them in servitude. - Ruled that neither Congress nor the Court has the powers to deal with private acts of acts of discrimination.

Name of Case: Korematsu v. United States Year: 1944

Constitutional Principle: - Equality - Rights of minority groups Why Decision is Important: - Ruled that the forcible relocation of Japanese Americans to Wartime Relocation Agency camps during World War II was legal.

Name of Case: Plessy v. Ferguson Year: 1896

Constitutional Principle: - Equality / Equal Protection - Rights of minority groups Why decision is Important: - Gave legal justification for racial segregation for different races were legal as long as those facilities were equal to one another. - Overturned in 1954 by Brown v. Board of Education.

Name of Case: Brown v. Board of Education Year: 1954

Constitutional Principle: - Equality / Equal Protection - Rights of minority groups Why Decision is Important: - Ruled that segregation in education creates inequality. - Overturned Plessy v. Ferguson and nullified the concept of "Separate but equal."

Name of Case: Gibbons v. Ogden Year: 1824

Constitutional Principle: - Federalism - Property rights and economic policy / - The judiciary Why Decision is Important: - Established the basis of congressional regulation of interstate commerce. - Reinforced the supremacy of national law over state law when the two conflict.

Name of Case: Worcester v. Georgia Year: 1832

Constitutional Principle: - Federalism -National power.- Separation of powers.- Equality. Why Decision is Important: - Stated that treaties between the United States government and Indian nations are the supreme law of the land. - Declared that the federal government, not the state, had exclusive jurisdiction over - Cherokee nation's territory; therefore, Georgia laws taking jurisdiction of Cherokee people and land were void. - President Jackson supported Georgia in defying this ruling, and Native American removal followed.

Name of Case: Wabash, St. Louis and Pacific RR v. Illinois Year: 1886

Constitutional Principle: - National power - Federalism Why Decision is Important: - The supreme Court forbade any state to set rates, even within its own borders, on railroad traffic entering from or bound for another state. This paved the way for the creation of the Interstate Commerce Commission in 1887.

Name of Case: Northern Securities Co. v. United States Year: 1904

Constitutional Principle: - National power Why Decision is Important: - Property rights and economic policy. - Federal suit brought as part of Theodore Roosevelt's trust-busting using Sherman Anti-trust Act. - Court in 5-4 decision ruled that the Northern securities Company was formed only to eliminate competition and ordered it to be dissolved.

Name of Case: Locher v. New York Year: 1905

Constitutional Principle: - Property rights and economic policy - Civil liberties Why Decision is Important: - Established that the Supreme Court has the power to oversee state regulations. - Ruled hat a New York law limiting baker's hours was unconstitutional because it interfered with workers' Fourteenth Amendment right to sell their labor to their employers.

Name of Case: Schechter Poultry Corporation v. United States Year: 1935

Constitutional Principle: - Separation of powers - Property rights and economic policy Why Decision is Important: - Placed limits on the ability of Congress to delegate legislative powers to President. - Narrowly defined interstate commerce. - Declared the New Deal's NRA unconstitutional.

Name of Case: Scott v. Sanford Year: 1857

Constitutional Principle: - The judiciary / Due Process - Equality Why Decision is Important: - Declared that slaves were property and that slaveholders could take them anywhere. Without risk of the slaves being freed. - Ruled that African American were not citizens. - Declared the Missouri Compromise unconstitutional; this decision was overturned by the Thirteenth and Fourteenth Amendment.

Name of Case: Tinker v. Des Moines Independent Community School District Year: 1969

Constitutional Principle: -Civil liberties - Freedom of Speech Why Decision is Important: - Ruled that certain kinds of nonverbal communication can be protected under the First Amendment.

Name of Case: Mapp v. Ohio Year: 1961

Constitutional Principle: - Avenues of representation / Due Process - Federalism Why Decision is Important: - Upheld the principle that population is the only acceptable basis for the apportionment of seats in a legislative body. - Established that the Supreme Court has cases when that reapportionment threatens voters rights.

Name of Case: Roe v. Wade Year: 1973

Constitutional Principle: - Civil liberties - Right to Privacy Why Decision is Important: -Ruled that state laws that criminalize abortion are unconstitutional.

Name of Case: United States v. Nixon Year: 1974

Constitutional Principle: - Federalism - Separation of powers - National Supremacy Why Decision is Important: - Limited the President's right to confidentiality. - Gave federal courts the right to decide when and how that confidentiality should be limited.

Name of Case: United States v. E.C. Knight Co. Year: 1895

Constitutional Principle: - National power Why Decision is Important: - Ruled that Congress has the right to protect trade and commerce against unlawful restraints and monopolies.

Name of Case: Marbury v. Madison Year: 1803

Constitutional Principle: -Federalism -Separation of powers / Checks and Balances - The judiciary / Judicial Review Why Decision is Important: - Established the Supreme Court's right of judicial review. - Strengthened the judiciary in relation to other branches of government.

Name of Case: New Jersey v. T.L.O. Year: 1985

Constitutional Principle: Equal Protection - civil liberties Search & Seizure Why Decision is Important: - Ruled that juveniles have the right to the same protection as adults against illegal search and seizure. - More clearly defined what constituted a legal search and seizure.

Name of Case: In Re Debs Year: 1895

Constitutional Principles: - National power Why Decision is Important: - Reinforced that the right of Congress to regulate interstate commerce extends to the commerce that is conducted by railroad and highway. - Ruled that the federal government has the right to intervene forcibly to eliminate monopolies in transportation of people, property, and mail.

The "wall of separation" that the Constitution maintains between church and state is contained in the a. free exercise clause. b. establishment clause. c. incorporation doctrine. d. community standards clause. e. due process clause.

The correct answer is B. In 1947, the Supreme Court held in Everson v. Board of Education that the wording of the First Amendment: "Congress shall make no law respecting an establishment of religion..." should be broadly applied to keep church and state separate. This "wall of separation" prohibits prayer is school and public funding for religious education.

When Congress authorized the President to eliminate individual elements from proposed legislation, it was known as a(an) a. presidential prerogative. b. line item veto. c. Executive decision. d. legislative veto. e. subpoenaed document.

The correct answer is B. In the past, Congress authorized the President to veto specific elements of proposed legislation without vetoing an entire bill. The intent was to help the President limit government waste by removing unnecessary spending attached to essential bills. However, the line item veto gives a great deal of power to the Executive branch and was ruled unconstitutional by the Supreme Court.

In the Federalist Papers, James Madison proposed that the states under the new Constitution would retain a. absolute sovereignty. b. residual sovereignty. c. supreme sovereignty. d. no sovereignty. e. virtual sovereignty.

The correct answer is B. Madison proposed that, instead of the absolute sovereignty of each state under the Articles of Confederation, the states would retain a "residual sovereignty" in all those areas that did not require national concern. The very process of ratification of the Constitution, he argued, symbolized the concept of federalism rather than nationalism. He said: "This assent and ratification is to be given by the people, not as individuals composing one entire nation, but as composing the distinct and individual states to which they respectively belong. The act, therefore, establishing the Constitution, will not be a national but a federal act."

Which of the following committees in the House of Representatives gets the greatest attention from lobbyists? a. Armed Services b. Appropriations c. Veterans' Affairs d. Judiciary e. Rules

The correct answer is B. Members of the Appropriations Committee receive greater attention from lobbyists because their committee provides funding for regulatory agencies and government contracts. Committee members receive large campaign donations from the health, defense, and energy industries.

Most of the actual work of Congress takes place a. on the House of Representatives floor. b. in the standing committees. c. in the caucuses. d. in the House Rules Committee. e. on the Senate floor.

The correct answer is B. Most of the detailed work of creating and modifying legislation takes place in the various standing committees. Unlike the Senate, little debate takes place on the floor of the House of Representatives.

In non-partisan elections for state or local government offices, a. political parties may not contribute to campaigns. b. ballots do not list party affiliations. c. most voters are independents. d. incumbent candidates run unopposed. e. candidates cannot be registered members of any political party.

The correct answer is B. Non-partisan elections simply remove the party labels from candidates. Candidates can maintain their private party affiliations, but their names will appear on the ballot without a party designation.

Which Supreme Court case established the principle of judicial review and the power of the Court to determine the constitutionality of legislative and executive acts? a. Gibbons v. Ogden b. Marbury v. Madison c. Dred Scott v. Sandford d. Brown v. Board of Education e. New York Times Co. v. Sullivan

The correct answer is B. Often called the most important decision in the history of the Supreme Court, Marbury v. Madison established the principle of judicial review and the power of the Supreme Court to determine the constitutionality of legislative and executive acts. The case arose from a political dispute in the aftermath of the Presidential election of 1800.

The Senate can introduce legislation related to any subject EXCEPT a. changing education policy. b. raising revenue. c. appropriating funds to agriculture. d. changing veterans' benefits. e. refocusing child labor laws.

The correct answer is B. Only the House of Representatives can introduce legislation related to raising revenue.

Which of the following does NOT actively lobby Congress? a. Labor unions b. Political parties c. Corporations d. Foreign governments e. Senior citizens

The correct answer is B. Political parties do not lobby Congress. Most interest group lobbies are created specifically to petition Congress to change or maintain specific government policies or laws.

Which of the following functions is NOT performed by the major political parties in the American system? a. Helping to connect the people with their government b. Increasing the diversity of viewpoints represented in the political arena c. Stimulate and focusing public discussion d. Promoting candidates and voter participation e. Helping resolve political conflicts and promoting national unity

The correct answer is B. Political parties in the American system tend to limit the expression of extreme views. Instead, the two parties tend to move towards the political center, causing more extreme views to be underrepresented.

The process of reapportionment takes place every a. two years matching the federal election cycle. b. ten years after the completion of the federal census. c. four years following the inauguration of a new President. d. six years prior to elections for the Senate. e. time the party controlling the state legislature changes.

The correct answer is B. The Constitution calls for reapportionment every ten years after the completion of the federal census. State legislatures are responsible for determining the new congressional districts.

The Constitution gives the power to "raise and support armies" to the a. President. b. Congress. c. Judiciary. d. Department of State. e. Department of Defense.

The correct answer is B. The Constitution gives the power to prepare for and declare war to the Congress and the power to execute war to the President. The Constitution gives no powers to any specific departments.

Which of the following would be an unconstitutional action by a state government? a. Signing an extradition treaty with another state b. Banning the sale of goods from a foreign country c. Regulating tobacco sales d. Setting a minimum age for the consumption of alcohol e. Regulating electricity usage by industry

The correct answer is B. The Constitution grants the federal government the exclusive right to negotiate treaties for trade and security. Banning goods from a given country would usurp federal authority and would be unconstitutional

According to the Constitution, who is the president of the Senate? a. Senate Majority Leader b. Vice President c. President d. President Pro Tempore e. Chaplain of the Senate

The correct answer is B. The Constitution names the Vice President as the president of the Senate and the Senate's presiding officer. However, other than breaking a tie, the role is mostly ceremonial.

Dred Scott v. Sandford (1857)

Slavery, Due Process, the Missouri Compromise In 1834, slave Dred Scott was purchased in Missouri and then brought to Illinois, a free (non-slave) state. His owner and he later moved to present-day Minnesota where slavery had been recently prohibited, and then back to Missouri. When his owner died, Scott sued the widow to whom he was left, claiming he was no longer a slave because he had become free after living in a free state. At a time when the country was in deep conflict over slavery, the Supreme Court decided that Dred Scott was not a "citizen of the state" so they had no jurisdiction in the matter, but the majority opinion also stated that he was not a free man.

The Writ of Habeas Corpus a. protects individuals from being held without being charged b. allows individuals to appeal capital sentences c. guarantees an individual's right of religious expression d. prevents the use of cruel and unusual punishment e. grants appellate jurisdiction to federal courts for Constitutional issues

The correct answer is A. A writ of habeas corpus demands that a prison official bring an inmate to the court to determine if he is imprisoned lawfully or if he should be released from custody. The Writ of Habeas Corpus was important enough to be part of the original body of the Constitution and has been an important limit on state abuses of power by the federal court system.

Which of the following are tools of congressional oversight used to evaluate governmental programs? I. General Accounting Office II. Congressional Research Service III. Congressional Budget Office a. I only b. II only c. III only d. I and II only e. I, II and III

The correct answer is A. The General Accounting Office is the primary watchdog for Congress. It studies how the federal government uses tax dollars, evaluates federal programs, audits federal expenditures, and issues legal opinions.

Texas v. Johnson (1989)

Flag Burning, Freedom of Speech In a political demonstration during the Republican National Convention in Texas, protesting the policies of the Reagan Administration and of certain corporations based in Dallas, Gregory Lee Johnson doused an American flag with kerosene and set it on fire. No one was hurt or threatened with injury, but some witnesses said they were seriously offended, and Johnson was charged and convicted with the desecration of a venerated object, in violation of the Texas Penal Code. In a split decision, the Supreme Court determined that Johnson's actions were symbolic speech protected by his First Amendment Rights.

The strongest factor contributing to an individual's political socialization tends to be a. family. b. peers. c. media. d. school. e. religion.

The correct answer is A. The family environment is the dominant factor in the political socialization of an individual. Children tend to adopt the same political perspectives as their parents as well as the same party affiliation.

What is the greatest expense of running a presidential election campaign? a. Travel b. Advertising c. Salaries d. Entertainment e. Polling

The correct answer is B. Advertising is by far the greatest expense of running a national campaign. Television, radio, and print ads usually account for more than half of total campaign spending.

The doctrine of "prior restraint" is most closely associated with a. religious tolerance. b. freedom of the press. c. liable speech. d. sexual lifestyles. e. political assembly.

The correct answer is B. Federal courts developed the doctrine of "prior restraint" to protect the press from preemptive censorship.

The governmental system of dividing power between a central government and several regional governments is known as a. democracy. b. federalism. c. republicanism. d. oligarchy. e. colonialism.

The correct answer is B. Federalism is characterized by a sharing of power between local and national levels. In America, power is divided between the states and the federal government in Washington, DC.

The elected position of President pro tempore of the Senate is almost always held by a. the Vice President. b. a senior senator of the majority party. c. a freshman senator of the majority party. d. the majority whip. e. the minority leader.

The correct answer is B. The Constitution provides that, in the absence of the President of the Senate (the Vice President), the President pro tempore will preside. The PPT is almost always the most senior Senator of the majority party. Because the PPT is usually also a committee chairman with a busy schedule, the Senate evolved the tradition of putting freshman senators in the presidential chair, which aids them in learning the procedures of their new workplace.

Impeachment is a a. loss of office by a government official due to criminal activities. b. conviction of misconduct brought against a government official. c. jail sentence given to a government official. d. criminal offense performed by a government official. e. charge of misconduct brought against a government official.

The correct answer is E. Impeachment refers only to a charge of misconduct. It does not, as is commonly thought, refer to conviction on such charges. As set forth in the Constitution, the House of Representatives must bring charges of misconduct by voting a bill of impeachment. The accused official is then tried in the Senate, with the Chief Justice of the Supreme Court presiding at the trial.

Which of the following Supreme Court cases infamously decided that a slave was not a citizen but property to be "used in subservience to the interests, the convenience, or the will of his owner"? a. Plessy v. Ferguson b. Mapp v. Ohio c. United States v. Klein d. Strauder v. West Virginia e. Dred Scott v. Sandford

The correct answer is E. In 1846, Dred Scott and his wife Harriet filed suit for their freedom in St. Louis Circuit Court. In 1857, the case went before the U.S. Supreme Court, which ruled that Scott must remain a slave. This decision contributed to rising tensions between the free and slave states prior to the Civil War.

All of the following are typical stances of American liberals EXCEPT a. support for abortion rights. b. concern for the rights of the accused. c. resistance to state involvement with religious institutions. d. support for international organization such as the United Nations. e. opposition to the expansion of governmental activity and regulation.

The correct answer is E. Liberals tend to see a larger role for the government than do traditional conservatives, particularly in the regulation of business and industry. All of the other answer choices form core beliefs of the American political left.

Which strategy has been most successful for congressional lobbyists? a. Grassroots mobilization b. Public information campaigns c. Testimony in Congress d. Endorsements of candidates for public office e. Financial contributions to political campaigns

The correct answer is E. Lobbyists for special interest groups have found campaign contributions to be the most direct and effective method to secure their objectives in Congress.

An example of an independent grant-making agency of the federal government is a. NASA. b. the Arctic Research Commission. c. the Committee on Indian Affairs. d. the Department of Education. e. the National Endowment for the Humanities.

The correct answer is E. NEH is an independent grant-making agency of the United States government dedicated to supporting research, education, preservation, and public programs in the humanities. The agency was established in 1963.

Which of the following was originally granted the power to choose United States Senators? a. Senatorial districts voters b. All state voters c. State electoral colleges d. State governors e. State legislatures

The correct answer is E. Senators were originally chosen by the state legislatures.

Those calling for the adoption of the Constitution presented all of the following arguments EXCEPT that Federalism would a. protect individual rights while allowing the government to work for the common good. b. help prevent tyranny. c. help prevent war between the states. d. allow the states to be a testing ground for new policies and programs. e. give the states greater sovereignty and independence.

The correct answer is E. The Federalists never promised greater powers for the states. The Constitution aimed to bind the states closer together for their mutual benefit. Each state would give up some of its sovereignty and independence in order to become part of a much greater whole

According the Supreme Court, something is obscene and is not protected as free speech if it contains which of the following elements? I. It is prurient in nature. II. It is completely devoid of scientific, political, educational, or social value. III. It violates local community standards. a. I only b. II only c. III only d. I and II only e. I, II, and III

The correct answer is E. The First Amendment to the Constitution does not protect speech that can be defined as obscene. The Supreme Court has defined a thing as obscene if it meets all of the criteria listed above. The emphasis on local standards means that obscenity laws may differ from state to state.

Most Americans believe that a. the needs of the individual should be subordinate to those of the community. b. everyone should enjoy equal opportunities for personal and material success. c. the government should provide free healthcare for all citizens. d. the government rather than industry should be responsible for national economic development. e. Congress rather than the President should determine foreign policy.

The correct answer is B. The Declaration of Independence states that all men should have the rights of "life, liberty, and the pursuit of happiness." Today, most Americans take these words to mean that every American should enjoy equal opportunities for personal and material success.

The Department of Commerce promotes a. domestic aggregate demand and supply. b. international trade and economic growth. c. fair prices and stable markets for producers and consumers. d. improved technology through research, development, and demonstration. e. better working conditions and improved relations between labor and management.

The correct answer is B. The Department of Commerce is primarily concerned with ensuring the competitiveness of the U.S. economy. The Department provides businesses with information, administers programs to protect U.S. firms from unfair international competition, and provides standards of measure for U.S. industry.

Which of the following is a component of the executive branch of government? a. Congressional Budget Office b. Department of Energy c. Library of Congress d. Federal Judicial Center e. U.S. Tax Court

The correct answer is B. The Department of Energy is one of the various departments associated with the executive branch of government. A and C belong to the legislative branch. D and E belong to the judicial branch.

According to the Supreme Court, First Amendment free speech protections do not apply to a. the desecration of the American flag. b. obscenity. c. protests, rallies, or strikes against the democratically elected government. d. sexual content acceptable by community standards. e. verbal or written attacks on public figures.

The correct answer is B. The First Amendment protects all forms of free speech except obscenity. The argument is that obscenity has no redeeming value and is non-political in nature. There is no need for it to be protected by the Constitution.

Over the last five presidential elections, women have tended to a. vote for Republican Party candidates. b. ignore military issues important to male voters. c. vote for Democratic Party candidates more often than men do. d. vote in fewer numbers than men do. e. register to vote in fewer numbers than men do.

The correct answer is C. In recent presidential elections, women have tended to vote for Democratic presidential candidates more often than men do.

The term "Reagan Democrats" has become a generic term for a. people who do not vote. b. third-party candidates. c. independent voters. d. swing voters in the Democratic party. e. swing voters in the Republican party

The correct answer is D. "Reagan Democrats" is the term used for those Democratic voters who voted for the Republican President, Ronald Reagan, in the 1980s. "Reagan Democrats" is often used today as a generic term for swing voters in the Democratic party.

Minority political parties historically have included all of the following EXCEPT a. farmer-laborer parties that embrace populist, working-class ideals. b. parties that focus on a single issue. c. parties of ideological protest, such as the Libertarian or Socialist parties. d. the Independent party. e. factions that have split off from one of the two major parties.

The correct answer is D. A candidate who is running on the Independent ticket, by definition, does not identify with any party. Historically, minority parties have been formed when groups have concluded that their interests have not been adequately served by either of the two major parties. Sometimes, the new party splinters from one of the major parties (E). This was the case with the American Green party, which split off from the Democratic party and nominated consumer and environmental advocate Ralph Nader as their presidential candidate in 2000. Often minority parties will focus on a single issue (B), such as the Right To Life party, which opposes abortion. The rise of populist movements that embrace working-class ideals is a recurrent theme in American politics, and has occasionally resulted in the formation of new political parties (A), such as the Progressive parties of 1916 (known as the Bull Moose party) and 1924.

A coherent set of values and beliefs about the purpose and scope of government describes an individual's a. political socialization. b. class consciousness. c. family history. d. political ideology. e. socioeconomic class.

The correct answer is D. A political ideology is an interrelated set of values and beliefs about role of government in society.

All of the following are true about the federal budget EXCEPT: a. Mandatory spending accounts for the majority of all spending. b. Discretionary spending must be decided each year by Congress and the President. c. Appropriation bills provide funds for the spending outlined by Congress and the President. d. Congressional oversight helps to ensure that programs are well managed and achieve their intended results. e. Spending for entitlement programs must be approved in each budget cycle.

The correct answer is E. Spending for entitlement programs such as Social Security, Medicare, and veterans' benefits is provided each year by permanent laws. This spending can only be changed if Congress passes and the President approves new legislation.

Shay's Rebellion demonstrated the weakness of a. the United States Constitution. b. federalism. c. colonial administration. d. the Articles of Confederation. e. slavery.

The correct answer is D. In Shay's rebellion, Massachusetts farmers rose up to protest their economic plight. The incident shook the confidence of many at the Constitutional Convention and shifted the opinions of many delegates towards favoring a more powerful national government.

The practice of submitting proposed legislation for approval by a popular vote is known as a a. plebiscite. b. gridlock. c. primary. d. referendum. e. recall.

The correct answer is D. In a number of states, legislation proposed by elected officials or the general public can become law through the referendum process. Typically, a number of signatures are required to place the referendum on the ballot and a simple majority for the referendum to pass. Although a plebiscite is also an appeal for public approval, the term has broader meaning of approval for the government as a whole and is not used in American government.

The basic principles of a free society include all of the following EXCEPT a. free market economy. b. personal and family responsibility. c. limited government. d. national income tax. e. rule of law.

The correct answer is D. Individual freedom and national parliamentary government are also basic principles of a free society. National income tax is not a basic principle or requirement for a free society. Income tax was introduced in 1799 in Britain, originally as a temporary measure to help fund the war against Napoleon. In 1861, income tax was introduced in the United States, as a temporary measure to help fund the Civil War.

Which of the following factors is likely to be the weakest indicator of an individual's political beliefs and opinions? a. Race b. Gender c. Age d. Intelligence e. Religion

The correct answer is D. Intelligent individuals can be of any race, religion, gender etc. Each of the other groups listed are the basis for formal and informal political organizations that foster like-minded political attitudes and beliefs.

Which of the following is a tactic often used by interest groups to influence the federal bureaucracy? a. Picketing federal office buildings b. Striking and work stoppages c. Protesting and civil disobedience d. Influencing political appointments e. Refusing to pay taxes

The correct answer is D. Interest groups often attempt to influence the appointment of department heads. For example, environmentalists and industry leaders vie with each other to influence the President's choice for head of the Environmental Protection Agency.

Congressional members who are finishing their final terms in Congress are known as a. conferees. b. cats and dogs. c. ranking members. d. lame ducks. e. sponsors.

The correct answer is D. Lame duck members of Congress find it difficult to meet their legislative objectives. They have limited political capital because they must leave Congress at the end of their term.

A congressional representative's efforts to help a constituent resolve a problem with the federal bureaucracy is known as a. sponsorship. b. franking. c. caucus. d. casework. e. lobbying.

The correct answer is D. Members of Congress view casework as a useful way to please constituents and win votes.

Which of the following Supreme Court cases addressed the right to privacy? a. Hawaii Housing Authority v. Midkiff b. Bowsher v. Synar c. Korematsu v. United States d. Bowers v. Hardwick e. Gibbons v. Ogden

The correct answer is D. Michael Hardwick was a gay man from Georgia charged with committing sodomy in his own home with a consenting adult. The court ruled that the Constitution does not explicitly grant the right for homosexuals to practice their lifestyle and that laws against sodomy were Constitutional.

Which of the following is the largest interest group in the United States? a. The American Medical Association (AMA) b. The National Rifle Association (NRA) c. The United Auto Workers (UAW) d. The American Association of Retired Persons (AARP) e. The National Association for the Advancement of Colored People (NAACP)

The correct answer is D. More than 35 million U.S. senior citizens are members of the American Association of Retired Persons (AARP), making it the nation's largest interest group.

All of the following are Constitutional checks on executive power EXCEPT a. the Senate must approve international treaties. b. the Senate must confirm judicial appointees. c. Congress has the power to impeach the president. d. the President cannot propose legislation. e. the Judiciary can declare Executive actions unconstitutional.

The correct answer is D. Much of the legislation enacted by Congress is proposed by the Executive. The other answer choices are constitutional checks that limit the powers of the Executive and protect the nation from tyranny.

Congressional oversight has led to each of the following EXCEPT a. forcing officials out of office. b. changing policies. c. providing statutory controls over the executive. d. overturning unjust laws. e. preventing fraud.

The correct answer is D. The broad jurisdiction of congressional oversight applies to all of the answer choices except (D). Overturning unjust laws is the responsibility of the judicial branch, and specifically the Supreme Court.

Which of the following would be most likely to support government involvement in the economy? a. Libertarians b. Conservatives c. Anti-Federalists d. Liberals e. Anarchists

The correct answer is D. The common definition of "liberal" ideology includes a belief that the government has an important role to play in the economy. The other ideologies given are opposed to government involvement in the economy.

Which of the following amendments was the only amendment to be ratified through the process of "ratifying conventions," not a vote in the state legislatures? a. 13th Amendment (abolishment of slavery) b. 21st Amendment (repeal of prohibition) c. 16th Amendment (allowing income tax) d. 2nd Amendment (right to bare arms) e. 18th Amendment (prohibition)

The correct answer is D. The first ten amendments to the Constitution (the Bill of Rights) were passed in a ratifying convention rather than through votes of the state legislatures.

Which of the following has the LEAST influence on the federal bureaucracy? a. Senate b. House of Representatives c. President d. Supreme Court e. Presidential cabinet

The correct answer is D. The judiciary has little influence on bureaucrats and generally attempts to remain impartial. The cabinet has the greatest amount of influence on the bureaucracy since its members head up the various executive departments.

The Supreme Court's current view of the Second Amendment is that a. state governments cannot regulate firearms. b. state and federal governments are free to regulate firearms in any way they see fit. c. the federal government cannot regulate firearms. d. citizens do not have the right to own normal militia weapons. e. citizens have the right to own normal militia weapons.

The correct answer is E. Currently citizens have the right to keep and bear normal militia weapons. The only Supreme Court case to deal with the right to bear arms was heard in 1939. In its ruling, the Court interpreted the amendment to refer to weapons that were necessary "to the preservation or efficiency of a well regulated militia."

Which of the following make it difficult for elected officials to control the federal bureaucracy? I. Career bureaucrats often place personal or departmental interests first. II. Career bureaucrats have their own areas of policy expertise. III. Congress and the President compete for influence with the bureaucracy. a. I only b. II only c. I and II only d. II and III only e. I, II, and III

The correct answer is E. Each of the factors listed make it difficult for elected officials to influence the federal bureaucracy.

Which of the following allow the President to influence the legislative process in Congress? I. The President's access to the federal bureaucracy allows quick and effective access to experts and information. II. The President has better access to, and influence on, public opinion than does the Congress. III. The threat of a presidential veto can change a bill while it is still in committee. a. I only b. I and II only c. I and III only d. II and III only e. I, II, and III

The correct answer is E. Each of these factors contributes to the President's ability to influence the legislative process. During the twentieth century, Congress lost much of its traditional power to drive the legislative agenda, giving rise to a "presidential program" determined by the Executive.

Which of the following is a standing committee of Congress? a. Rights of Women b. Minorities in Education c. Social Security d. Welfare e. Education and the Workforce

The correct answer is E. Education and the Workforce is a standing committee of the House of Representatives. The committee's jurisdiction includes child labor, labor standards and statistics, and work incentive programs

Which of the following is NOT a basic function of the Congress? a. Representation b. Investigation and scrutiny c. Legislation d. Financial control e. Enforcement

The correct answer is E. Enforcement is the responsibility of the executive branch, not the legislative branch, of the federal government.

Regents of the U. of California v. Bakke (1978)

Affirmative Action, Equal Protection In the early 1970s, the medical school of the University of California at Davis devised a dual admissions program to increase representation of disadvantaged minority students. Allan Bakke was a white male who applied to and was rejected from the regular admissions program, while minority applicants with lower grade point averages and testing scores were admitted under the specialty admissions program. Bakke filed suit, alleging that this admissions system violated the Equal Protection Clause and excluded him on the basis of race. The Supreme Court found for Bakke against the rigid use of racial quotas, but also established that race was a permissible criteria among several others.

Korematsu v. United States (1944)

Japanese Internment, Equal Protection After Pearl Harbor was bombed in December 1941, the military feared a Japanese attack on the U.S. mainland and the American government was worried that Americans of Japanese descent might aid the enemy. In 1942, President Franklin D. Roosevelt signed an executive order forcing many West Coast Japanese and Japanese Americans into internment camps. Fred Korematsu, a Japanese American, relocated and claimed to be Mexican-American to avoid being interned, but was later arrested and convicted of violating an executive order. Korematsu challenged his conviction in the courts saying that Congress, the President, and the military authorities did not have the power to issue the relocation orders and that he was being discriminated against based on his race. The government argued that the evacuation was necessary to protect the country and the federal appeals court agreed. Korematsu appealed this decision and the case came before the U.S. Supreme Court. The Court agreed with government and stated that the need to protect the country was a greater priority than the individual rights of the Japanese and Japanese Americans.

McCulloch v. Maryland (1819)

State Taxes, National Supremacy The U.S. government created the first national bank for the country in 1791, a time during which a national bank was controversial due to competition, corruption, and the perception that the federal government was becoming too powerful. Maryland attempted to close the Baltimore branch of the national bank by passing a law that forced all banks that were created outside of the state to pay a yearly tax. James McCulloch, a branch employee, refused to pay the tax. The State of Maryland sued McCulloch saying that Maryland had the power to tax any business in its state and that the Constitution does not give Congress the power to create a national bank. McCulloch was convicted and fined, but he appealed the decision. The Supreme Court determined that Congress has implied powers that allow it to create a national bank, even though the Constitution does not explicitly state that power, and that Maryland's taxing of its branches was unconstitutional.

A bill loaded with special projects designed to please home constituents at the expense of the federal taxpayer is known as a. pork barrel legislation. b. courtesy legislation. c. pay-as-you-go legislation. d. germane legislation. e. appropriation legislation.

The correct answer is A. A bill loaded with benefits for home constituents is known as "pork barrel legislation."

According to the Constitution, the outcome of a presidential election shall be decided by the House of Representatives in the case that a. a single candidate does not get a majority of electoral votes. b. there is a virtual tie between the two leading candidates in the popular vote. c. there is a virtual tie between the two leading candidates in the popular vote and a vote in the Senate fails to select a winner. d. a candidate wins the popular vote but loses the Electoral College vote. e. a "spoiler" candidate is determined to have taken votes away from one of the other candidates.

The correct answer is A. According the Constitution, if a single candidate does not get a majority of votes in the Electoral College, the outcome of the presidential election shall be decided by the House of Representatives.

America's form of government is best described as a. republican. b. aristocratic. c. parliamentary. d. unitary. e. egalitarian.

The correct answer is A. Although based on the principles of democracy, America's government is republican—the people elect officials to represent them in government.

Which of the following interest groups might make use of litigation rather than lobbying to advance their interests? a. Prisoners b. Unions c. Corporations d. Senior Citizens e. Educators

The correct answer is A. Although organizations such as the American Civil Liberties Union lobby for prisoners' rights, litigation remains the most viable tactic for the majority of inmates.

The annual federal budget process is initiated by the a. President. b. House of Representatives. c. Senate. d. Appropriations Committee. e. Ways and Means Committee.

The correct answer is A. Although the Constitution does not require the President to present an annual budget, in 1921 the Budget and Accounting Act became law and lay the foundation for the modern budget process, which includes the President's budget. More recently, the Congressional Budget and Impoundment Control Act of 1974 established a timetable for the annual budget process, which is kicked off each year by the Presidential budget submission. The Budget Act specifies that the President's budget should be presented to the Congress on or before the first Monday in February. This generally coincides with the timing of the President's annual State of the Union Address before the Congress.

A party's presidential and vice-presidential candidates are formally chosen and its national platform is adopted a. at the national convention. b. through a vote and direct primaries. c. through a lottery system. d. by state party organizations. e. by members already in government.

The correct answer is A. Although the party national conventions are little more than pep rallies before the presidential campaign, some substantive work is done. In addition to formally selecting the party's presidential and vice-presidential candidates, the convention debates and adopts a party platform that contains the party's commonly agreed principles and objectives.

The most powerful position in the House of Representatives is the a. Speaker. b. Vice President. c. Majority Whip. d. Minority Leader. e. President Pro Tempore.

The correct answer is A. At the beginning of each new House session, a Speaker is chosen to preside over the body. Traditionally, the Speaker is the leader of the majority party in the House. Although the Vice President is also president of the Senate, he has no powers in the House. The President Pro Tempore is also a Senate leader.

An increased focus on national rather than local issues on radio, television, and other mass media has a. caused Americans from different regions to share similar ideas and perspectives. b. caused a divergence of political beliefs and attitudes across America. c. reduced public interest in foreign affairs. d. limited the influence of the media on domestic politics. e. pushed the political parties away from the political center.

The correct answer is A. Before the advent of radio and other mass media, much political news focused on local issues. Larger audiences have caused national issues to be treated in the media. Today, most Americans watch some form of national news. This has caused a convergence of ideas and perspectives and the development of a more national rather than regional identity among the public.

Quick and often humorous comments made by candidates for the media are know as a. sound bites. b. free media. c. sound checks. d. positive ads. e. contrast ads.

The correct answer is A. Candidates attempt to control the way they are presented in the media through the use of sound bites. Sound bites are tailored comments that can be easily adapted for presentation in the media. Ideally, these short quotes should be funny and convey as concisely as possible the candidate's intended message.

The role of caucuses in Congress is to a. promote group interests through legislation, policy, and pressure on the agencies. b. measure existing support for a bill before the House of Representatives makes a binding vote on it. c. muster support before an election cycle. d. bind members to vote for or against legislation prior to the full House of Representatives vote. e. block the passage of legislation by denying the opposition a simple majority.

The correct answer is A. Caucuses are informal organizations of individual congressional representatives with like interests or constituencies. Members work together to promote the interests of the groups they represent through legislation, policy, and pressure on government agencies. Caucuses may represent everything from agriculture to tourism and often cross party lines.

Powers unambiguously granted to the Congress by the Constitution are known as a. enumerated powers. b. implied powers. c. resulting powers. d. inherent powers. e. delegated powers.

The correct answer is A. Congressional powers explicitly outlined in the Constitution are known as "enumerated powers."

Spending by the national committees of the political parties to support the election of congressional candidates is known as a. coordinated spending. b. logrolling. c. hard money spending. d. matching funds. e. clean money.

The correct answer is A. Coordinated spending has been a target of advocates for campaign finance reform. This is because the limits on contributions to national parties are less stringent than those on individual candidates. Coordinated spending has been an important tool for both parties.

Most African Americans support the Democratic Party because a. of its support for the advancement of the civil rights cause. b. urban voters tend to support Democrats and many African Americans live in cities. c. of Democratic support for looser immigrations laws. d. of its tough stand on "law and order" issues. e. it was the party that ended slavery.

The correct answer is A. Democrats gained great popularity among African Americans for their support of the Civil Rights Movement. Although the Republican President Eisenhower signed the first Civil Rights Act in 1957, the Republican Party has been perceived as less supportive of the civil rights cause.

Which of the following statements about the Atomic Energy Act of 1946 is true? a. It transferred the control of atomic energy programs from military to civilian hands. b. It was signed into law by President Dwight D. Eisenhower in a ceremony on the aircraft carrier Kitty Hawk. c. It provided funds for the establishment of an Operations Office in Princeton, New Jersey. d. It provided the funding for the Manhattan Project, which resulted in the creation of the world's first atomic bomb. e. It established guidelines for the commercial sale of nuclear energy from nuclear power plants.

The correct answer is A. Following the 1945 atomic bombing of Japan that brought an end to World War II, the Atomic Energy Act was signed into law by President Harry S. Truman on August 1, 1946. In addition to transferring control of atomic energy programs from military to civilian hands, it provided funds for the establishment of Operations Offices, the first of which was opened in Chicago, Illinois in 1947. The Atomic Energy Act gave the Atomic Energy Commission extraordinary power to carry out its mission: to guide the use of atomic energy by the United States, not only for military purposes, but also (as stated in the Act) for the promotion of world peace, improvement of the public welfare, and furtherance of competition of free enterprise.

During the colonial period, royal representatives possessed veto power over acts of the colonial legislatures. Because of wide opposition to this practice, most state constitutions at first gave no veto power to governors. Over time, however, governors gained veto powers, in some cases extending to particular items in appropriations bills. This suggests that state governors have? a. made significant gains in legislative power with the adoption of the line-item veto. b. used the veto in the way that was intended by the framers of the Constitution. c. used the veto to overturn legislation that had been popular with the citizens of their states. d. expanded their veto power to include the pocket veto. e. done a more effective job of governing than royal representatives did in colonial times.

The correct answer is A. Historically, the veto has always been a powerful political tool. Expanding the power to include a line-item veto on appropriations bills marks an increase in the power of the governors. In early democratic systems, the veto was problematic; legislative bodies in which each member had veto power over the decisions of the whole were found to be highly susceptible to bribery and corruption. The Constitution balances the presidential veto by giving the Senate the ability to overturn the veto with a two-thirds vote. There is no information given to indicate that answers B through E are either true or false.

Which of the following issues was first decided by the Supreme Court? a. Individual states cannot encroach on the powers of Congress to regulate interstate commerce. b. A woman has the right to chose whether or not she terminates her pregnancy. c. Segregation based on race is unconstitutional. d. Capital punishment is unconstitutional because it represents "cruel and unusual punishment." e. The line-item veto is unconstitutional because it is not a power enumerated in the Constitution.

The correct answer is A. In Gibbons v. Ogden (1824), the Supreme Court struck down a New York law that sought to control trade and transport between New York and other states by granting a steamship monopoly to a firm operating on New York's waterways. The Constitution gives Congress the right to regulate trade between the states. All of the other cases are from the twentieth century.

Which of the following Supreme Court cases involved the principle of an implied right to personal "privacy" in the Constitution? a. Roe v. Wade b. Mapp v. Ohio c. Fullilove v. Klutznick d. Marbury v. Madison e. Gibbons v. Ogden

The correct answer is A. In Roe v. Wade, the Supreme Court ruled that the 9th and 14th Amendments granted rights that were broad enough to encompass a woman's right to decide for herself whether or not she terminates her pregnancy.

Which of the following is associated with greater political participation? a. Having high socioeconomic status b. Being a minority c. Having strong religious beliefs d. Having low socioeconomic status e. Being a college student

The correct answer is A. Individuals of with high socioeconomic status are more likely to participate politically. The other answer choices tend to indicate lower levels of political participation.

One of the objects of "Jim Crow" laws was to a. disenfranchise Americans of color. b. limit soft-money contributions. c. empower southern blacks. d. protect the economies of southern states. e. desegregate southern schools.

The correct answer is A. Jim Crow laws were designed to prevent people of color in the south from threatening the traditional political and economic dominance of southern whites. These laws enforced segregation in practically every aspect of daily life including the workplace, education, and public facilities.

Federal district court judges are a. nominated by the President and confirmed by the Senate. b. nominated by the Chief Justice and confirmed by the Senate. c. elected by standing judges of the same circuit. d. nominated by a senator and confirmed by the entire Senate. e. elected by the citizens of the district in which they will serve.

The correct answer is A. Judges of the Federal Court system—Supreme Court justices, court of appeals judges, and district court judges—are nominated by the President and confirmed by the United States Senate. This process is outlined in the Constitution and neither the public nor the judiciary itself can be formally involved in the process.

The view that the Supreme Court should interpret the Constitution or laws to advance social goals is known as a. judicial activism. b. judicial restraint. c. judicial review. d. judicial conservatism. e. judicial liberalism.

The correct answer is A. Judicial activism is the philosophy that judges in the federal court system can and should seek to address important social issues that elected officials have not addressed. This is typically done by carefully choosing to hear cases in which a decision will right a perceived social wrong. The opposite judicial philosophy is known as judicial restraint.

Why are Senate races typically more competitive than House of Representatives races? a. A single party rarely dominates an entire state, and gerrymandering cannot effect the statewide vote. b. Higher campaign spending by incumbent House representatives leads to more one-sided races than in the Senate. c. The issues debated in Senate campaigns tend to be more volatile than those that dominate House races. d. Incumbent senators have little ability to pass legislation that will win them support from their constituents. e. Senators have less latitude to depart from their party platforms, making it difficult to please their constituents.

The correct answer is A. Senate races are competitive because a senator must appeal to a much larger constituency than a representative in the House. House representatives are often able to use gerrymandering to create congressional districts that grant them a healthy majority of supporters.

Which of the following best explains the concept of "judicial restraint"? a. The Supreme Court (and lesser courts) should not interpret the judges' own philosophies or policy preferences into the law but should continue the law whenever reasonably possible to avoid second-guessing the legislative and executive branches. b. The Supreme Court (and lesser courts) should avoid handing down decisions that are likely to result in either an appeal of the decision, or the overturning of an earlier decision. c. The Supreme Court shall not hear certain kinds of cases without first conducting its own investigation into the causes of action that led to the case being filed. d. The judicial system should strive for ideological balance, with equal numbers of conservative and liberal judges on the bench in each state. e. In criminal trials, federal courts should impose the lightest possible sentence, and grant probation whenever possible, in order to avoid overburdening the federal prison system.

The correct answer is A. Judicial restraint demands that the courts should not read the judges' own philosophies or policy preferences into the Constitution and laws. It holds that because judges have no popular mandate to make legislation, they should defer to the decisions of the elected branches, as long as those policymakers stay within the limits of their powers as provided by the U.S. Constitution and the constitutions of the states.

Philosopher John Locke, in his 1690 work Civil Government (second treatise), advised separation of government power between a. the legislative and executive branches. b. the judicial and executive branches. c. the judicial and legislative branches. d. the executive branch and tax collectors. e. the executive, legislative, and judicial branches.

The correct answer is A. Locke expanded on the concepts of Aristotle, who favored a mixed government composed of monarchy, aristocracy, and democracy. In his 1656 work Oceana, James Harrington brought these ideas up-to-date, proposing a modern system based on the separation of power.

An agreement between two or more members of Congress for reciprocal support in passing unrelated legislation is known as a. logrolling. b. filibustering. c. pork barreling. d. gerrymandering. e. reapportioning.

The correct answer is A. Members of Congress often support each other based on a simple promise of "I'll vote for 'x' if you vote for 'y.'" This practice of making these informal promises of support is called logrolling. Party leaders and whips keep track of vote debts between party members and use them to gain support when important party-sponsored legislation must be passed.

Minorities other than African Americans have had the greatest success advancing their civil rights through a. litigation. b. constitutional amendments. c. consumer boycotts. d. legislative initiatives. e. mass media advertising.

The correct answer is A. Minorities have had great success advancing their causes through the courts. A host of organizations provide legal support for individuals and groups who wish to challenge what they feel are discriminatory laws.

How has the expansion of cable and satellite television affected the President's ability to influence American public opinion? a. It has increased the President's power to influence public opinion. b. It has not affected the President's ability to influence public opinion. c. It has reduced the President's power to influence public opinion. d. It has sharply decreased the President's power to influence public opinion. e. It has eliminated the President's ability to influence public opinion.

The correct answer is A. Modern media technology has made it possible to follow the President's every movement twenty-four hours a day. The President, therefore, has greater access to the public and a greater ability to influence their opinions than at any point in the nation's history.

The concept embodied in the Constitution of separating the legislative, executive, and judicial functions of government was promoted by which philosopher? a. Baron de Montesquieu b. David Hume c. John Locke d. Sir Isaac Newton e. Sir Francis Bacon

The correct answer is A. Montesquieu argued for the specialization and balancing of governmental powers. Like Locke, Montesquieu's thinking greatly influenced America's emerging democracy.

Over which of the following is there broad consensus in American public opinion? a. The institutional basis of American democracy b. The application of American values to social problems c. The incorporation of religious or ethical values in public policy d. The priorities of American foreign policy e. The role of the government in the private sector

The correct answer is A. Most Americans agree on the institutional basis of American democracy: the Constitution, three branches, and checks and balances that are characteristic of American government. However, opinions about how to use the government to solve specific problems are more diverse.

The Constitution allows the federal government to do all of the following EXCEPT a. change state boundaries. b. tax imports. c. tax exports. d. overturn state laws. e. impose taxes.

The correct answer is A. None of the original states would have accepted that the federal government have the power to change state boundaries. They were wary of excessive federal power.

What has been the constitutional basis for challenges to campaign contribution limits? a. Free speech b. Due process c. Right to assembly d. Right to privacy e. Right to petition the government

The correct answer is A. Opponents of limits on campaign contributions have argued that such restrictions limit an individual's right to free speech. They view campaign contributions as a form of free speech.

Which of the following does NOT define a power relationship between states and a federal government? a. Pluralism b. Devolution c. Dual federalism d. States' rights e. Confederation

The correct answer is A. Pluralism is a philosophy of society and government that recognizes diversity as engine for social change and progress. It does not deal with issues of state and federal government.

The term that refers to occasional historic shifts of public opinion and voter concerns which either undermine or enhance a party's traditional base of support is a. realignment. b. plurality. c. conservatism. d. delegation. e. census baiting

The correct answer is A. Realignment is generally applied to national elections that clearly shift the majority and minority status of the two U.S. major political parties, or that replace one of the two major political parties with one that previously had been a third party. Realignment may be based on many factors, such as the reaction to party positions on a critical issue of national concern (as was the case with the slavery issue in the 1860s), credit or blame for handling a national crisis (such as the Great Depression of 1929), or substantial changes in the demographic composition of the voting populace.

Voter turnout usually increases given which of the following circumstances? a. There is a binding referendum on the ballot. b. There is an advisory referendum on the ballot. c. The incumbent candidate is widely predicted to win in a landslide. d. One of the candidates withdraws from the race on the eve of the election. e. There is an outspokenly religious candidate on the ballot.

The correct answer is A. Studies have shown that having a binding referendum on the ballot usually increases voter turnout. An advisory referendum on the ballot does not have this same effect. Both the incumbent being widely predicted to win in a landslide and a candidate withdrawing on the eve of the election are factors that could cause voter turnout to decrease. The presence of an outspokenly religious candidate on the ballot has not been shown to have a correlative effect on voter turnout.

Which of the following is the "lame duck" amendment to the Constitution? a. 20th Amendment (limiting service of elected officials) b. 5th Amendment (cannot be tried twice for the same offense) c. 18th Amendment (prohibition) d. 3rd Amendment (soldiers quartered in one's home) e. 7th Amendment (right to jury trial)

The correct answer is A. The "lame duck" amendment is the popular name for the 20th Amendment to the Constitution, ratified on February 6, 1933. It is designed to limit the time that elected officials can serve after the general election in November. This amendment provides, among other things, that the terms of the President and Vice President end at noon on January 20, the terms of Senators and Representatives end at noon on January 3, and the terms of their successors then begin.

Which of the following Amendments to the United States Constitution were repealed? I. The 18th Amendment (Prohibition) II. The 4th Amendment (Search and Seizure) III. The 19th Amendment (The Right to Vote) a. I only b. II only c. I and II only d. II and III only e. I, II, and III

The correct answer is A. The 18th Amendment—prohibiting the production, sale, and transportation of alcoholic beverages—was repealed with the 21st Amendment to the Constitution. It was the only Amendment to the Constitution to be repealed.

Which Senate committee writes the annual legislation releasing federal funds to government agencies, departments, and organizations? a. Appropriations b. Government Affairs c. Finance d. Ways and Means e. Budget

The correct answer is A. The Appropriations committee drafts the funding legislation that keeps government programs financed. It is the Senate's largest committee and one of its most powerful.

All bills to raise government revenue must originate in the a. House of Representatives. b. Senate. c. Senate Budget Committee. d. Department of the Treasury. e. Internal Revenue Service.

The correct answer is A. The Constitution gives only the House of Representatives the right to introduce legislation to raise government revenues. Departments and agencies may provide input on legislation under consideration, but they cannot introduce legislation. The Senate cannot introduce bills to raise government revenue.

The Constitution gives Congress the power to a. declare war. b. command U.S. armed forces. c. appoint ambassadors. d. negotiate treaties. e. nominate federal judges.

The correct answer is A. The Constitution grants Congress the exclusive right to declare war. Over time, the Executive branch has eroded this power by entering combat without a formal declaration of war.

To become a Supreme Court Justice, an individual nominated by the President must receive the "consent" of the a. Senate. b. House of Representatives. c. Attorney General. d. Judiciary Committee. e. Supreme Court.

The correct answer is A. The Constitution grants the Senate the power to confirm all executive appointments including those to the judiciary. A successful nominee serves with the "consent" of the Senate.

Which of the following is responsible for the conservation of America's natural resources? a. Department of the Interior b. Department of Justice c. Department of State d. Department of the Treasury e. Environmental Protection Agency

The correct answer is A. The Department of the Interior is responsible for most federally owned lands including those of the National Parks Service. It is also responsible for protecting wildlife through the Fish and Wildlife Service. Although the Environmental Protection Agency plays an important role in protecting the environment, it does not have the broad powers and responsibilities of the Department of the Interior.

A debate on which one of the following issues would be most likely to involve a discussion of the Establishment clause of the Constitution? a. Prayer in public schools b. Abortion c. Gun registration d. Drug enforcement policy e. Rural poverty and its causes

The correct answer is A. The Establishment clause of the First Amendment to the Constitution states that "Congress shall make no law respecting establishment of religion" and adds "or prohibiting the free exercise thereof." The latter clause is known as the Free Exercise clause. Some tension exists between the two clauses due to their potentially conflicting mandates, with some scholars arguing that the original goal of the framers regarding this point has never been known.

Which of the following is an example of a clientele agency? a. Farm Credit Administration b. Department of Defense c. Department of Justice d. Department of State e. Joint Chiefs of Staff

The correct answer is A. The Farm Credit Administration is considered a clientele agency. A clientele agency is typically set up by Congress to satisfy an important and politically active segment of society. It also represents the seizing of territory by Congress, which maintains nominal control over the agency, not the Presidency.

Among other things, the Federal Reserve Board a. conducts the nation's monetary policy and supervises and regulates banks. b. coins money and issues government bonds. c. administers the Selective Service System and maintains the National Guard. d. maintains America's petroleum and other strategic stockpiles. e. oversees the protection of federal lands and natural resources.

The correct answer is A. The Federal Reserve Board was created to provide America with a safe, flexible, and stable monetary and financial system. The Board supervises and regulates banks and controls the money supply by changing the discount rate it charges on its loans to banks. The Board does not coin money or issue bonds—that is the work of the Treasury Department.

The United States Constitution grants the right to a. not be tried twice for the same offense. b. a trial by jury in all civil and criminal cases. c. vote for all citizens of the age 21 or older. d. reasonable health and medical care. e. be indicted by a grand jury.

The correct answer is A. The Fifth Amendment of the Bill of Rights protects citizens from being tried twice on the same charge.

In Plessy v. Ferguson (1896), the Supreme Court upheld segregationist laws, arguing that the protections of the Constitution a. did not prohibit segregation where the races were separate but equal before the law. b. did not apply to apply since African Americans were not considered citizens. c. applied only to federal and not state law. d. did not include equality for all citizens regardless of color. e. did not apply to laws predating the Civil War.

The correct answer is A. The Plessy decision is famous for establishing the "separate but equal" standard that was overturned in Brown v. Board of Education. The Court ruled that as long as all races were equally responsible for following the law, segregation was constitutional. Later, in Brown, the Court ruled that "separate is never equal."

According to law, the President of the United States must be a. at least 35 years old. b. a college graduate. c. wealthy enough to afford a campaign. d. unconnected to a religion. e. a lifelong resident of the United States.

The correct answer is A. The President of the United States must be at least 35 years old, according to the Constitution. Other requirements are that the person must be a native-born American citizen and at least 14 years a resident of the United States.

In the mid-term elections following a President's inauguration, his party tends to a. lose seats in Congress. b. gain seats in Congress. c. lose House of Representatives seats but gain Senate seats. d. lose Senate seats but gain House of Representatives seats. e. lose control of Congress.

The correct answer is A. The President's party tends not to gain seats in Congress during the mid-term elections. The mid-term elections under President George W. Bush were an exception. The last time that a President's party gained seats in the Senate was in 1982, and no President had gained backing in the House since the Civil War.

The state officer typically responsible for overseeing federal elections at the state level is the a. Secretary of State. b. Attorney General. c. State Auditor. d. Governor. e. State Treasurer.

The correct answer is A. The Secretary of State is appointed by the Governor to supervise the election process. For example, in the controversial 2000 presidential election, it was Florida's Secretary of State Katherine Harris (appointed by Governor Jeb Bush) who was responsible for making sure that Florida's election was free, fair, and impartial.

The fact that the Senate alone is entrusted with confirming presidential appointments and ratifying treaties, while the House of Representatives takes precedence in money matters, is an example of a. checks and balances. b. laissez-faire politics. c. senatorial privilege. d. fiduciary duty. e. fiscal responsibility.

The correct answer is A. The Senate is entrusted with confirming appointments in order to preclude the President from appointing someone who has not been reviewed by another body. Similarly, a treaty made by the President with another country is not considered in force until it has been ratified, or approved, by the Senate. These powers are given exclusively to the Senate in order to counterbalance the House's taking precedence in money matters.

Ambassadors, as well as embassy and consular staff, are all employees of the a. State Department. b. Defense Department. c. Department of the Interior. d. Agency for International Development. e. Congress of the United States.

The correct answer is A. The State Department handles America's international relations and employs ambassadors and all other American embassy and consular staff abroad.

The Supreme Court has exclusive jurisdiction in a dispute between a. two states. b. two multinational corporations. c. a foreign and an American citizen. d. a foreign and an American corporation. e. political parties.

The correct answer is A. The Supreme Court is the only judicial body that may hear disputes between states. A small number of cases are brought directly to the Supreme Court each year without going through lower levels of the federal or state court system. Cases that qualify are known as "original jurisdiction" cases since they are defined in the Constitution.

Which of the following are prohibited under Eighth Amendment protections against "cruel and unusual punishment"? a. Execution of the mentally retarded b. Execution for capital crimes c. Use of voluntary prison labor d. Execution by hanging e. Solitary confinement

The correct answer is A. The Supreme Court ruled that executing a mentally retarded individual for any crime would constitute "cruel and unusual punishment" as prohibited under the Eighth Amendment. The Court decided that a "national consensus" believed that a retarded person should not be put to death. In his minority opinion, Justice Scalia argued that any convicting jury embodied a "community standard" of what was or was not "cruel and unusual punishment."

In addition to holding the right of succession, the Vice President is the presiding officer of the a. Senate. b. House of Representatives. c. Department of State. d. United Nations. e. National Security Council.

The correct answer is A. The Vice President serves concurrently with the President. In addition to holding the right of succession, the Vice President is the presiding officer of the Senate. The 25th Amendment, adopted in 1967, amplifies the process of Presidential succession. It describes the specific conditions under which the Vice President is empowered to take over the office of President if the President should become incapacitated.

The lowest court in the federal system is the a. district court. b. superior court. c. court of appeal. d. circuit court. e. Supreme Court.

The correct answer is A. The district courts are the trial courts of the federal system. There are 94 federal courts across the country dealing with civil and criminal matters.

Which of the following statements best reflects the elite theory of American politics? a. American politics is dominated by a small elite who is responsible for most of the important policy decisions. b. Public policies emerge from compromises reached among competing groups. c. The American political arena is made up of individuals, many of whom are striving to be part of an elite. d. Public policy arises directly from the will of the majority, as expressed through voting patterns. e. Many U.S. senators come from wealthy families.

The correct answer is A. The elite theory of American politics holds that a small moneyed elite exercises dominant control over the most important policy decisions. B reflects the leading theory of American politics, the pluralistic theory, while D reflects the majoritarian theory of American politics, considered obsolete, which holds that public policy arises directly from the will of the majority. C and E do not pertain to any established theory of American politics.

Political action committees (PACs) and lobbyists generally contribute more money to political parties than to individual candidates' campaign funds, for the main reason that a. there is a strict legal limit on the amount that can be contributed to an individual candidate, but no such limit on contributions to a political party. b. contributions to political parties are tax-deductible, whereas contributions to individual politicians are not. c. political parties always spend more than individual candidates during a campaign. d. PACs may be Democrat, Republican, or neither. e. individual candidates are prohibited from accepting campaign funds directly from PACs.

The correct answer is A. The limit that a PAC may contribute to an individual candidate's campaign fund is $5,000. There are no limits on contributions to political parties, as long as the funds are used for "party-building" efforts. While PACs are often either Democrat or Republican (D), party affiliation does not dictate the size of campaign contributions.

The primary function of the Cabinet is to a. assist the President in executing laws and making policy decisions. b. prepare legislation for the Congress. c. serve as a link between the President and the American public. d. translate presidential policies to work at the state and local levels. e. meet as the National Security Council.

The correct answer is A. The members of the Cabinet are the secretaries of the executive departments. It is their responsibility to help the President make policy and execute the laws of the nation.

Which of the following correctly states the order of succession to the presidency following the death or removal of the President? a. Vice President, Speaker of the House, President Pro Tempore of the Senate, Secretary of State b. Vice President, Secretary of State, Speaker of the House, President Pro Tempore of the Senate c. Vice President, President Pro Tempore of the Senate, Speaker of the House, Secretary of State d. Vice President, Speaker of the House, Secretary of State, President Pro Tempore of the Senate e. Vice President, Secretary of State, President Pro Tempore of the Senate, Speaker of the House

The correct answer is A. The order of succession to the Presidency has changed several times through history. The current order was determined by The Presidential Succession Act of 1947. The Speaker of the House was made third in line, the majority leader of the Senate (President Pro Tempore) fourth, with cabinet members following in the order that their departments were created.

The power of judicial review grants the Supreme Court the ability to a. overturn or modify the enforcement of laws that violate the Constitution. b. evaluate all judicial appointments prior to Senate consent proceedings. c. edit bills under consideration prior to enactment. d. consider all state laws for constitutionality prior to their enactment. e. rate the effectiveness of all federal laws and regulations.

The correct answer is A. The primary power of the Supreme Court is its ability to challenge the constitutionally of state and federal laws. If a law is declared unconstitutional by the Supreme Court, it cannot be fully enforced until it is revised.

Which of the following statements accurately describes the role of the Senate Committee on Appropriations? I. Allocates federal funds to government agencies, departments, and organizations on an annual basis II. Drafts budget resolutions that determine spending levels III. Proposes legislation related to banking and financial institutions a. I only b. II only c. III only d. I and II e. I, II, and III

The correct answer is A. The primary responsibility of the Appropriations Committee is to allocate funds to the numerous governmental agencies and departments. Statement II describes a responsibility of the Budget Committee. Statement III describes a responsibility of the Banking Committee.

An example of an implied constitutional right is the right to a. privacy. b. free speech. c. free assembly. d. keep and bare arms. e. right to vote.

The correct answer is A. The right to privacy is an implied right since it is not mentioned in the Constitution. However, the 9th Amendment states that "the enumeration in the Constitution, of certain rights, shall not be construed to deny or disparage others retained by the people." In other words, citizens can enjoy certain implied rights not mentioned in the Constitution as long as they are in keeping with the spirit of the Constitution.

Which of the following is a difference between the House of Representatives and the Senate? a. There are more than four times as many House Representatives as there are Senators. b. Debates tend to last longer in the Senate than in the House of Representatives. c. There are many single-party committees in the House of Representatives, but most committees in the Senate are bipartisan. d. The House of Representatives has the exclusive power to ratify treaties. e. Seniority is less important in the House of Representatives than in the Senate.

The correct answer is A. There are 100 Senators and 437 House Representatives. The Senate, not the House of Representatives, has the exclusive power to ratify treaties. Seniority is equally as important in the House of Representatives as in the Senate. All committees in both houses are bipartisan.

Normally, a bill that is tabled by a congressional committee will a. not be acted upon. b. be dealt with following the recess. c. be amended before being sent back to Congress. d. be vetoed by the President. e. not be amended prior to resubmission.

The correct answer is A. When a committee tables a bill, it effectively kills the bill; the bill will not be acted upon.

Most states have a. bicameral legislatures. b. unicameral legislatures. c. professional full-time legislatures. d. term limits for legislators. e. campaign spending limits for legislators.

The correct answer is A. With the exception of Nebraska, all state legislatures are bicameral.

In a popular election, a "spoiler" candidate is one who a. uses his own personal wealth to outspend the other candidates, winning the election as a result. b. tips the balance between two leading candidates by attracting a minority of voters who otherwise might have voted for one of the leading candidates. c. participates in the general election, withdrawing from the race before the electoral college election. d. does not expect to gain office but runs mainly as a form of protest against another candidate. e. runs mainly for the purpose of trying to expose alleged vote fraud.

The correct answer is B. A spoiler candidate is one who attracts just enough votes in a close election to tip the balance between two leading candidates. The 2000 Green Party candidacy of Ralph Nader is considered a prime example of a spoiler campaign, since it split the Democratic vote between Nader and Albert Gore, clearing the way for Republican George W. Bush to capture the election.

After a bill has been introduced in the House of Representatives or the Senate, and referred to a conference committee and a subcommittee, the usual next step is a. the bill is submitted to the rules committee for consideration. b. a public hearing is held to explore different points of view on the measure. c. the text of the bill is published in the Congressional Record. d. amendments to the bill are proposed and voted on in a markup session. e. the bill is submitted for the President for approval or veto.

The correct answer is B. After a bill has been introduced in the House or Senate and referred to a conference committee and a subcommittee, it is usual for the subcommittee to hold a public hearing on the measure so that different points of view can be brought to light and discussed. This phase is followed by a "markup session," in which amendments to the bill are put forth and voted on. Following the bill's passage by both the House and the Senate, it is submitted to the President for him to sign, or veto, or let pass into law unsigned.

After hearings are complete, a congressional subcommittee will usually determine a bill's future in final deliberations known as a. an open door session b. a markup session c. a closed door session d. a joint resolution session e. a full house session

The correct answer is B. After hearings are concluded and all relevant information is collected, the subcommittee will meet in a "markup" session to determine what recommendation will be forwarded to the full committee, or if the bill should be "tabled" so that no action is taken.

A brief filed by an entity that is not a party to a lawsuit, but claims to have an interest in its outcome, is known as a. a writ of certiorari brief. b. an amicus brief. c. a prosecutor's brief. d. defendant's brief. e. plaintiff's brief.

The correct answer is B. An "amicus brief" is often filed in Supreme Court cases where the implications of a ruling will likely affect many citizens other than those who are parties to the case. For example, civil rights groups often file amicus briefs in civil rights cases to provide the justices with their arguments and relevant information.

The United States Constitution was opposed by which group? a. Abolitionists b. Royalists c. Nationalists d. Smaller states e. Federalists

The correct answer is B. Anti- Federalists supported the crown's right to control America and would oppose any American Constitution.

Which of the following explain why the founders of the Constitution decided upon a bicameral rather than a unicameral legislature? I. It balanced the interests of low- and high-population states by giving smaller states a voice in the Senate that their low populations would not grant them in the House of Representatives. II. Either legislature could block legislation approved by the other, thereby strengthening the American system of checks and balances. III. The House of Representatives was granted responsibility for domestic affairs while the Senate specialized in America's relations with the rest of the world. a. I only b. I and II only c. II and III only d. III only e. I, II, and III

The correct answer is B. At the Constitutional Convention, delegates from the heavily populated states wanted proportional representation while delegates from the less populated states feared that their voices would not be heard. The Constitution created a House of Representatives based on proportional representation and a Senate in which each state had equal representation. The bicameral system also strengthened the system of checks and balances because passage of legislation required cooperation between the two legislatures. Although Senate ratification of treaties and the House initiation of tax legislation seem to imply international and domestic focuses respectively, these considerations were not part of the Constitution.

Since the Second World War, presidential approval ratings have generally a. risen gradually until mid-term elections then fallen between election cycles. b. declined gradually after inauguration through to the end of the administration. c. increased gradually from inauguration through to the end of the administration. d. fallen dramatically after mid-term elections then risen. e. risen dramatically after inauguration through to the end of the administration.

The correct answer is B. Barring a war or other major event, presidential approval ratings tend to fall gradually over time as groups of voters become less satisfied with the President. After election, most Presidents enjoy an approval rating that is actually higher than the support they enjoyed in the presidential election. However, each of a President's actions tends to alienate some supporters and over time his popularity tends to fall.

The power of a cabinet member within a department may be limited if a. the economy is faltering and unemployment and prices rise. b. career administrators do not fall in line with their politically appointed supervisor. c. they are assigned to a department concerned with domestic affairs. d. their constituents demand too many political favors. e. other cabinet members have stronger political ties with the President.

The correct answer is B. Cabinets are often put together for political rather than purely administrative reasons. Bureaucrats who have built their careers within an agency may have different beliefs and expectations than their politically appointed bosses. As a result, it is difficult for even skilled cabinet secretaries to affect radical change in the departments they run.

The function of conference committees is to a. gauge public opinion through testimony from interest groups and other witnesses. b. resolve the differences between existing House of Representatives and Senate versions of a bill. c. determine political strategy and adopt platform for the congressional session. d. offer concessions to win support for a bill from the President and his cabinet. e. join together state party leaders prior to the general election.

The correct answer is B. Conference committees join together members of the appropriate House of Representatives and Senate standing committees to resolve conflicts between versions of a bill passed by each chamber. Once a compromise bill is agreed upon, it is sent to the House of Representatives and then to the Senate for approval.

Which of the following contradicts the idea of a pluralistic America? a. Faith-based organizations that campaign to criminalize abortion b. Lack of ethnic and religious diversity among representatives in Congress c. Ethnic and religious lobbies that can influence law makers d. Religious communities that have chosen to live apart from other Americans e. The printing of ballots and voter information in foreign languages

The correct answer is B. Despite progress made during the Civil Rights Movement, the membership of Congress does not accurately represent American diversity. Pluralism is the belief that society should be governed by elites who represent the interests of the many ethnic, religious, and social groups that comprise modern society.

The primary purpose of the Americans with Disabilities Act (ADA) of 1990 was to a. establish a commission to study the extent to which architectural barriers prevented access to public buildings. b. consolidate the nation's disability laws and provide for strong federal enforcement of a strengthened disability rights mandate. c. establish the National Council on Disability (NCD) as an independent federal agency. d. request from the General Accounting Office an estimate of the costs of workplace accommodations and of removing architectural, transportation, and communication barriers. e. replace the term "handicapped" with the term "disabled" in general use.

The correct answer is B. Prior to the passage of the ADA, a patchwork of local, state, and national laws and regulations combined to define and carry forward the mandate of combating discrimination against the disabled and protecting their full citizenship rights under the Constitution. The first federal government commission of type A was authorized by Congress in 1965. After nearly two decades of governmental inquiry into the problems faced by disabled Americans, Congress moved to establish the National Council on Disability as an independent federal agency in 1984 (C). In 1989, the NCD requested from the General Accounting Office an estimate of the costs of accommodating the disabled in the workplace and of avoiding or removing barriers to equal access with regard to public buildings, transportation, and communication networks (D). The ADA strengthened the nation's disability rights mandate by consolidating the nation's scattered disability laws into a single federal code, and provided for federal enforcement to ensure widespread compliance.

The doctrine of applying amendments to the Bill of Rights to states and localities is known as a. promulgation. b. incorporation. c. devolution. d. confederation. e. nullification.

The correct answer is B. Rights which have been "incorporated" apply to states as well as the federal government. For example, with the exception of "trial by jury," the due process protections of the Bill of Rights did not originally apply to the states. It wasn't until the 14th Amendment that they were incorporated into the state governments. Other rights such as freedom of speech and the right to bear arms were originally conceived as incorporated rights.

A larger percentage of the Hispanic immigrants who have arrived in the United States since 1990 identifies with the Democratic party. Among Asian immigrants who arrived during the same time period, a high percentage identifies with the Republican party. Which of the following statements gives the most reasonable explanation for the difference in party identification between Hispanic and Asian immigrants? a. The Democratic party actively recruits Hispanic immigrants but does not actively recruit Asian immigrants. b. While both Asian and Hispanic immigrants experience various types of discrimination, Hispanic immigrants are more likely to experience economic discrimination. c. Many Hispanic immigrants have lived under Communist rule, while most Asian immigrants come from established democracies. d. The President makes more diplomatic missions to Asia than to Latin America in an average year. e. Many Hispanics come from politically conservative countries in Latin America, where religion plays a significant role in political life.

The correct answer is B. Studies show that while Asian and Hispanic immigrants have to face many of the same hurdles with regard to ethnic-based discrimination in the United States, Asians face less economic discrimination compared with Hispanics. The greater economic difficulty faced by Hispanics, in theory, would cause them to gravitate more toward the Democratic party than the Republican party.

Currently, the most important form of media for candidates who want to become elected officials is a. the Internet. b. television. c. radio. d. newspapers. e. magazines.

The correct answer is B. Television media coverage often dictates how a candidate is perceived, and television coverage often "makes or breaks" a candidate. The first television broadcast of a political convention came in 1940, with an audience of 100,000 viewers. By the 1950s, television was reaching one-third of America's households. The two parties spent $3.5 million on television ads during the 1952 campaign, with the Republicans continuing to outspend the Democrats by a large margin. The 1960 Kennedy-Nixon debates clinched the crucial role of television in modern campaigning.

The original purpose of the Bill of Rights was to limit the powers of a. state governments. b. the federal government. c. both state and federal government. d. the British crown. e. the courts.

The correct answer is B. The Anti-Federalists feared that a strong federal government would result in tyranny. They insisted that the Constitution include a list of things that the government "couldn't do." With a Bill of Rights limiting the power of the federal government, the Anti-Federalists agreed to support the Constitution.

Which of the following were weaknesses of the Articles of Confederation? I. The government lacked the power to compel states to honor national obligations. II. The government had no powers of taxation. III. It failed to provide the authority to negotiate peace with England. a. I only b. I and II c. I, II, and III d. II and III e. III only

The correct answer is B. The Articles of Confederation provided for a weak national government without powers of taxation or the ability to force reluctant states to live up to their obligations. However, it did provide for a unified foreign policy and the negotiation of the Treaty of Paris in 1783.

The number of representatives from each state to serve in the House of Representatives is determined by the a. State Legislatures; state legislatures then define congressional voting districts acting within constraints set down by Congress and the Supreme Court. b. Congress; state legislatures then define congressional voting districts acting within constraints set down by Congress and the Supreme Court. c. Department of Commerce's decennial census; federal district courts then determine the congressional districts within the states. d. Department of State's decennial census; federal district courts then determine the congressional districts within the states. e. Senate; state legislatures then define congressional voting districts acting within constraints set down by Congress and the Supreme Court.

The correct answer is B. The Congress fixes the size of the House of Representatives and the procedure to apportion House seats among the states. State legislatures are then responsible for defining the voting districts that will elect congressional representatives. The Supreme Court and Congress try to limit redistricting and reapportionment that appears to be enacted for purely political ends. Although Congress uses data from the Department of Commerce's census, the Department itself plays no role in the process.

Which two British Acts, passed in 1764, caused American colonists for the first time to organize protests against the injustices of British rule, and sparked the rallying cry "no taxation without representation"? a. The Tea Act and the Stamp Act b. The Sugar Act and the Currency Act c. The Quartering Act and the Stamp Act d. The Declaratory Act and the Townshend Acts e. The Stamp Act and the Currency Act

The correct answer is B. The Sugar Act was the first British Act specifically aimed at raising money from the American colonies for the Crown. The Act increased duties on non-British goods shipped to the colonies. The Currency Act prohibited the colonies from issuing their own currency, angering many colonists. Participants in a town meeting in Massachusetts coined the phrase "no taxation without representation," and by the end of the year had organized a significant boycott of imported British goods in protest.

Which of the following is NOT part of the Executive Office of the President? a. National Economic Council b. Bipartisan Commission on the Future of Medicare c. Office Management and Budget d. National Security Council e. Office of National Drug Control Policy

The correct answer is B. The agencies that make up the Executive Office of the President work for the "institution" of the Presidency rather than for the sitting President. There is nothing "bipartisan" about them. Agencies within the EOP help the President to follow developments within an increasingly complicated government. Their employees conduct research and inform the President.

The attempted impeachment of President William J. Clinton in 1998 failed because a. the motion to impeach passed neither the House of Representatives nor the Senate. b. the motion to impeach passed the House of Representatives but failed to pass the Senate. c. the motion to impeach passed the Senate but failed to pass the House of Representatives. d. there was no historical precedent for impeaching a President. e. the House of Representatives found Clinton guilty of perjuring himself but decided that it was not an impeachable offense.

The correct answer is B. The articles of impeachment against President Clinton passed the House of Representatives, but stalled in the Senate, where a small but influential cadre of Republicans felt that the perjury charges that had been brought against Clinton in the aftermath of the Monica Lewinsky affair did not constitute the type of "high crimes and misdemeanors" that would warrant his removal from office.

The "excesses of democracy" decried by the founders of the Constitution pertained to the a. carnival atmosphere that prevailed during elections. b. tyranny of the majority over the minority. c. expansion of the vote to women and minorities. d. large deficits caused by pork barrel legislation. e. destruction of social and class distinctions.

The correct answer is B. The founders feared that a fully empowered majority would always threaten the individual rights of the minority. As Jefferson said, "An elective despotism was not the government we fought for." For this reason, the founders decided upon a federal republic with checks and balances and constitutional rights that prevented a majority from exercising excessive power over a helpless minority.

Which department has the largest number of career government professionals working abroad? a. Department of Transportation b. Department of Defense c. Department of State d. Department of Commerce e. Department of Justice

The correct answer is B. The greatest number of Americans career government professionals working abroad work for the Pentagon and the Department of Defense. Many Americans also work within embassy and diplomatic missions abroad for the Department of State. The Central Intelligence Agency (CIA) and National Security Agency (NSA) employ many Americans overseas.

In 1998, the Supreme Court struck down the presidential line-item veto law, declaring it unconstitutional, because a. it violated the congressional representatives' right to free speech. b. Congress did not have the constitutional authority to hand that power to the President. c. a President cannot have influence in the legislative process. d. the Bill of Rights demands that laws be passed in their entirety. e. the role of lobbyists would be reduced in Washington.

The correct answer is B. The passage of the Line Item Veto Act of 1996 strongly increased the power and authority of the executive branch over federal spending decisions. William J. Clinton was the first President to receive line-item veto power over the federal budget, beginning in January, 1997. In 1998, the Supreme Court overturned the law, ruling that Congress did not have the authority to hand that power to the President. A constitutional amendment would be required to put the line-item veto into effect.

The responsibility of the President to appoint federal judges when vacancies occur on the bench is referred to as a. judicial review. b. judicial selection. c. presidential privilege. d. judicial activism. e. preferential treatment.

The correct answer is B. The principle of judicial review (A) arose from the 1803 Supreme Court case Marbury v. Madison, and is the doctrine by which the courts have the ability to strike down laws that are deemed unconstitutional. Presidential privilege (C) is the name given to the President's power to withhold certain information, particularly with regard to advice he has received.

One key result of the War Powers Act of 1973 was that it a. provided the executive branch full authority to declare and wage war. b. placed limitations on the initiation of military interventions by the executive branch. c. brought the Supreme Court into the process of deciding whether or not to go to war. d. allowed for a general vote by the people of the United States on the decision to commit troops. e. weakened the role of Congress in military decisions by placing more authority with the Joint Chiefs of Staff.

The correct answer is B. The revival of congressional activity during the Vietnam War led to the War Powers Act of 1973, which placed limitations on the initiation of military interventions by the executive branch. Congress specified that such actions could not continue for more than 60 days without precipitating congressional review. If Congress did not approve the executive action, the President would have to withdraw troops. This legislation forced the executive branch to obtain congressional approval of significant military operations.

Spending determined by the number of qualified recipients and their legally determined need is called a. supplemental spending. b. deficit spending. c. entitlement spending. d. essential spending. e. discretionary spending.

The correct answer is C. Entitlement programs such as Medicare, Medicaid, Social Security, and Veterans' benefits have no set spending limits. Total spending on entitlements is determined only by the number of citizens qualifying for benefits and their needs.

All of the following are included in the Bill of Rights EXCEPT the right to a. a speedy and public trial. b. be informed of the right to have a lawyer present during interrogation. c. be indicted by a grand jury for capital crimes. d. petition the government for a redress of grievances. e. bear arms.

The correct answer is B. The right to be informed of the right to have a lawyer present during interrogation , as well as the right to remain silent to avoid self-incrimination following an arrest, both stemmed from Miranda v. Arizona, the historic 1966 case in which Supreme Court Chief Justice Earl Warren, writing for the majority, ruled that interrogating a suspect who has not been made aware of his rights under the Constitution undermined the privilege against self-incrimination guaranteed by the Fifth Amendment. All of the other answers are rights found in the Bill of Rights.

The principle that federal law must be followed even when it conflicts with state law is derived from the a. establishment clause. b. supremacy clause. c. due process clause. d. the necessary and proper clause. e. guaranty clause.

The correct answer is B. The supremacy clause declares that the Constitution "shall be the supreme law of the land" and that all state laws shall be subordinate to it.

The class of independent voters who do not vote according to party affiliation, but who typically have a broader range of concerns than single-issue voters, are known colloquially as a. Copperheads. b. Mugwumps. c. Indies. d. goo-goos. e. free voters.

The correct answer is B. The term "mugwump" began as an eastern Native American word for "chief." It gained political currency during the 1880 presidential campaign, during which many Republicans became disenchanted with candidate James G. Blaine, and switched sides to ally themselves with the Democratic contender, Grover Cleveland. The press called these party-switching voters "little mugwumps." In modern parlance, "mugwumps" refers to the mass of voters who do not identify strongly with either the Republican or the Democratic parties, but who are liable to vote for either in an election. Many political races have been won or lost based on a candidate's ability to sway the opinions of the mugwumps.

In the last 30 years, voter turnout among American 18- to 24-year-olds has been a. steadily increasing. b. steadily declining. c. up and down, mostly up. d. up and down, mostly down. e. basically holding steady.

The correct answer is B. U.S. Census Service statistics show that voter turnout in the 18- to 24-year-old bracket has been steadily declining over the last 30 years. The general voter turnout has also declined, but at a slower rate.

According to The Federalist Papers, in which of the following ways did the Articles of Confederation allow the majority to overrule the minority? I. A majority of the states could prevent the passage of new legislation to strengthen the confederacy. II. Nine of thirteen states, rather than a majority of the people, could control foreign affairs. III. The popular election of a powerful executive threatened the less populous states a. I only b. II only c. I, II and III d. II and III e. III only.

The correct answer is B. Under the Articles of Confederation, foreign relations could be controlled by a block of nine or more states voting together. A single state could block legislation, making it very difficult for a stronger confederation to develop. There was no provision for a powerful executive.

Why are unfunded mandates so popular for Congress? a. Unfunded mandates improve relationships between representatives and their constituencies. b. Representatives can satisfy constituents without being obligated to find funding. c. Most representatives have served at the state level and wish to return favors. d. State agencies are more efficient and flexible than the federal bureaucracy. e. State problems are best solved at the state level.

The correct answer is B. Unfunded mandates are laws passed by Congress that require the states to act at their own expense. While these measures are very unpopular with state officials, representatives under pressure from constituents benefit politically from this passing-of-the-buck.

A representative who obtains a government project or appropriation for his district, yielding specific benefits to himself and the district (such as patronage positions, increased employment, or public spending), is said to be practicing a. carpet bagging. b. soft money tactics. c. pork-barrel politics. d. logrolling. e. Reaganomics.

The correct answer is C. A "pork barrel" spending measure is one that brings a government project or appropriation to a political district, yielding benefits to the district and its political representative. The Reconstruction-era term "carpet bagging" in its modern usage refers to the seeking of political or economic gain in a region by nonresidents of the region. Logrolling refers to the process by which legislators trade votes in order to secure favorable action on projects of interest to them. Reaganomics is the name given to a loose set of conservative economic theories championed by President Ronald Reagan during his term in office, and which remain popular in Republican ideology.

An impeached President will be removed from office when a. two-thirds of the House of Representatives vote for his removal. b. the special prosecutor obtains a conviction before the Senate. c. two-thirds of the Senate vote for his removal. d. the Chief Justice of the Supreme Court signs a removal order forwarded from the House of Representatives. e. the President is found guilty by a twelve person jury of "high crimes or misdemeanors."

The correct answer is C. A two-thirds majority in the Senate removes a sitting President who has been impeached by the House of Representatives. Although the Chief Justice presides over the trial in the Senate, he has no special powers.

To negotiate a complex and politically sensitive trade treaty, a President will often a. defer to the Senate. b. use the War Powers Act. c. seek "fast-track" authority from the Congress. d. issue an executive order. e. defer to the Department of Commerce.

The correct answer is C. Fast-track authority allows the President to negotiate complex deals with foreign nations that would be difficult in the politicized environment of the Congress. While fast-track authority greatly facilitates the negotiation process, Congress retains its constitutional power to ratify treaties. The President must submit completed agreements to Congress for approval before they become binding.

In which of the following cases did the Supreme Court rule that police must inform suspects of their rights at the time of their arrest? a. Marbury v. Madison b. Bowsher v. Synar c. Miranda v. Arizona d. Lochner v. New York e. Ferguson v. Skrupa

The correct answer is C. In Miranda v. Arizona, the Supreme Court ruled that, at the time of their arrest, suspects must be informed of their right to remain silent and to have an attorney. These rights are commonly referred to as "Miranda Rights."

Which of the following may happen to a bill after it has been introduced in the House of Representatives or the Senate and referred to a committee? I. The committee chairman may refer the bill to the subcommittee that has jurisdiction over the bill's subject matter. II. In rare cases, the bill may be discharged from the committee and placed on the floor calendar. III. After being referred to a subcommittee, the bill may get passed to the President to be signed into law. IV. In a case where the subcommittee has made significant changes to a bill, the bill may be assigned a new number. V. The bill may never get considered and simply die in the committee. a. I and III b. III, IV, V c. I, II, IV, V d. I, II, III, and V e. I, II, III, IV, and V

The correct answer is C. After a bill has been introduced and referred to a committee, the committee chairman usually refers the bill to the subcommittee that has jurisdiction over the bill's subject matter (I). In rare case, the bill is discharged from further consideration by the committee and placed on the floor calendar (II). Committees are able to consider only a fraction of the bills that are referred to them; bills that do not receive consideration die in the committee (V). In a case where the subcommittee has made significant changes to the bill, the bill is given a new number and is called a "clean" bill (IV), usually bearing the name of the chairman of the subcommittee as its sponsor. After being referred to a conference subcommittee the bill is usually subject to a hearing and mark-up session—it is never passed immediately to the President (III).

Which of the following statements about the Securities and Exchange Commission is true? a. It was created by the Securities Act of 1933, also known as the "truth in securities" law. b. It replaced the self-regulatory organizations (SROs) that had previously governed the activities of the securities markets. c. It requires reporting of financial information by companies with publicly traded securities. d. It oversees the activities of the American gold and silver markets. e. It was created by the Investment Advisers act of 1940.

The correct answer is C. After the Securities Act of 1933 outlawing fraud and requiring companies to disclose financial and other important information prior to the sale of securities, the Securities Exchange Act of 1934 was passed, establishing the Securities and Exchange Commission. The SEC works in conjunction with the markets' Self-Regulatory Organizations to enforce the nation's securities laws. The SEC requires regular reporting of financial information by companies with publicly traded securities.

Which factor best explains why individuals might change opinions that they have expressed in a survey? a. Survey bias can cloud an individual's true viewpoint. b. Individuals may not be able to read the survey. c. People often do not have underlying attitudes to anchor their opinions. d. Media coverage and agendas change rapidly. e. Family pressures can change the answers that an individual gives on a survey.

The correct answer is C. An individual is more likely to change his or her opinion if that opinion is not based on some deeper attitude or principle.

The right of the President to avoid disclosing sensitive information, particularly with regard to advice he has received, is called a. executive order. b. presidential pardon. c. presidential privilege. d. right of insolubility. e. presidential indemnification.

The correct answer is C. As an example, President William J. Clinton invoked presidential privilege to avoid disclosing the details of the 177 pardons and commutations he granted to various people in the final days of his administration. Clinton's move was upheld in federal district court, prompting criticism from those who felt the judge's ruling reflected an extreme interpretation of presidential privilege.

Which of the following can stop an executive order from being carried out? I. Congress may pass a bill to nullify the order. II. The Supreme Court can rule that the order is unconstitutional. III. Three-fourths of the states can vote to nullify the order. a. I only b. III only c. I and II only d. II and III only e. I, II, and III

The correct answer is C. Congress can pass a "nullification" bill that declares the executive order no longer effective. Alternatively, any federal court may declare the executive order unconstitutional and prevent it from being carried out. No part of an executive order can be illegal or unconstitutional. The states have no power to interfere with an executive order.

All of the following statements about interest groups are correct EXCEPT: a. They may represent business, labor, agriculture, professionals, ethnic or religious groups, public interest groups, or single-issue groups. b. They are often criticized for placing special interests above the public interest. c. They are prohibited from contributing campaign funds or lobbying the legislature in an attempt to influence public policy. d. They may file court appeals or challenges in order to stimulate action on an issue. e. They may organize public relations campaigns, confrontations, or demonstrations in an attempt to sway public opinion on an issue.

The correct answer is C. Contributing campaign funds and lobbying the legislature are the two main ways that interest groups, which may represent any of the groups described in A, attempt to achieve their policy objectives. Other methods include filing court appeals and challenges (D), and organizing public relations campaigns, confrontations, or demonstrations. Legislators are routinely criticized for putting the special interests of lobbying groups above the public interest (B).

In which of the following theories of federalism does the national government and the states have authority in strictly defined and mutually exclusive areas of policy? a. Creative federalism b. New federalism c. Dual federalism d. Unified federalism e. Cooperative federalism

The correct answer is C. Dual federalism was the prevailing form of federalism in the United States until the 1900s. States' rights gradually eroded as the federal government extended its powers to regulate businesses, the economy, and civil rights.

All of the following aspects of Congress affect the legislative process EXCEPT a. parties. b. committees. c. hours. d. rules. e. staff.

The correct answer is C. Each of the answer choices directly affects the legislative process. There are no mandated hours during which Congress must be open or closed.

Which category of interest groups has been the strongest and most effective in lobbying Washington? a. Ideological interest groups b. Religious interest groups c. Economic interest groups d. Government interest groups e. Single-issue interest groups

The correct answer is C. Economic interest groups were the first to organize and to begin the systematic lobbying of Washington officials. Economic interest groups include industrial lobbies and professional organizations such as the American Medical Association

James Madison proposed to deal with the threat of political factions by a. encouraging greater party membership. b. providing civic education to the electorate. c. dividing authority among different governmental institutions. d. strengthening the Executive and Judiciary at the expense of the Congress. e. prohibiting the formation of political parties.

The correct answer is C. In the Federalist Papers, Madison argues that a bicameral legislature is less susceptible to the violence of factions—particularly when the two bodies are as different as the House and Senate. He believed that the popular nature, shorter terms, and greater turn-over in the House would naturally put it at odds with the more patrician Senate. He felt it would be difficult for a faction to obtain a dangerous two-thirds majority in both houses.

Which of the following philosophers believed that the responsibility of the state was to protect life, liberty, and property? a. Voltaire b. John Stuart Mill c. John Locke d. Benjamin Franklin e. Baron de Montesquieu

The correct answer is C. John Locke's Second Treatise of Government outlines his version of the social contract underlying just government. He believed that individuals surrendered a degree of personal autonomy so that the government could protect life, liberty, and property.

The concept of "popular sovereignty" as incorporated in the Constitution was first promoted by which philosopher? a. Thomas Hobbes b. Jean Jacque Rousseau c. John Locke d. John Stewart Mill e. Jeremy Bentham

The correct answer is C. Locke argued that the public should have the power to remove a legislature that didn't act for the public good. Hobbes' conception of popular sovereignty was much more limited than that which is incorporated in the Constitution.

When selecting a member of his cabinet, a President might hope to accomplish which of the following? I. Reward supporters II. Strengthen links with Congress III. Establish a new executive department IV. Strengthen links with key interest groups a. I and II only b. III and IV only c. I, II, and IV only d. I, III, and IV only e. I, II, III, and IV

The correct answer is C. Only an act of Congress can create an executive department. Cabinet selections are highly political, and the President uses his appointments to strengthen existing relationships or to build new ones.

Which group of Americans were the last to receive the right to vote? a. African Americans b. Women c. Americans between the ages of 18 and 21 d. Native Americans e. Asian Americans

The correct answer is C. Prior to 1971, only citizens 21 years or older could vote. The 26th Amendment extended the right to vote to all Americans of 18 years of age and older.

Senators serve terms of a. four years. b. twelve years. c. six years. d. two years. e. five years.

The correct answer is C. Senators serve six-year terms. One-third of the Senate stands for election every two years.

Those who believe judges should not interfere in policy, and should defer to the elected branches of the federal government, would be advocates of a. original jurisdiction. b. judicial oversight. c. judicial restraint. d. judicial activism. e. prior restraint.

The correct answer is C. Supporters of judicial restraint contend that the Constitution granted policy-making powers exclusively to the elected branches of the federal government. Therefore, the Judiciary should not second-guess policies promoted by the Congress or the Executive.

Which of the following factions supported stronger state powers and a weaker national government? a. Royalists b. Federalists c. Anti-Federalists d. Nationalists e. Whigs

The correct answer is C. The Anti-Federalists feared a tyrannical federal government that would erode state powers. Federalists wanted a strong federal government.

The gains in combating racial discrimination in the housing market achieved by the passage of the Civil Rights Act of 1964 were undermined somewhat by a. "Jim Crow" laws, which remained legally in force throughout much of the South. b. the court-ordered busing of white students into African American neighborhoods to attend school. c. racist practices in the real estate market such as blockbusting and redlining, which continued illegally in many areas following passage of the Act. d. the involvement of organized crime syndicates in controlling real estate prices in neighborhoods designated for racial integration. e. the fact that President John F. Kennedy had been unable to convince the Senate to pass the Act in its original form, prior to his assassination.

The correct answer is C. The Civil Rights Act was proposed by President Kennedy in 1963, and signed into law, in a stronger version, by President Lyndon B. Johnson in 1964. This in no way hindered the efficacy of the Act, the most significant civil-rights legislation since Reconstruction (E). The practice of busing to achieve integration in public schools was based on various state court rulings following passage of the Act, but it did not have the effect described in (B). The practices of blockbusting (by which real estate investors spread fear that an influx of minorities into a neighborhood was going to lower property values, thereby inducing residents to sell their homes at below-market rates) and redlining (by which mortgages, home loans, and insurance were denied to residents of low-income or minority neighborhoods) continued illegally after passage of the 1964 Act and undermined its effectiveness in some areas.

Which of the following would most likely board a foreign ship suspected of drug trafficking at a U.S. port? a. Navy b. Marines c. Coast Guard d. Army Amphibious Unit e. Immigration and Naturalization Service

The correct answer is C. The Coast Guard is the lead federal agency for maritime drug interdiction and shares lead responsibility for air interdiction with the U.S. Customs Service. As such, it is a key player in combating the flow of illegal drugs to the United States. The Coast Guard's mission is to reduce the supply of drugs from the source by denying smugglers the use of air and maritime routes in the Transit Zone, a six million square mile area, including the Caribbean, Gulf of Mexico and Eastern Pacific.

Article I of the Constitution grants all legislative powers of the federal government to a. a subcommittee of the thirteen original states. b. a Congress divided into three chambers. c. a Congress divided into two chambers. d. a system of courts overseen by state governors. e. an Executive surrounded by a cabinet of advisors.

The correct answer is C. The Constitution grants legislative power to Congress, which is divided into two chambers: the Senate and the House of Representatives.

The Constitution provides for a national census every a. 1 year and a redistribution of House of Representatives seats according to state legislators. b. 2 years and a redistribution of House of Representatives seats according to school districts. c. 10 years and a redistribution of House of Representatives seats according to population shifts. d. 50 years and a redistribution of House of Representatives seats according to minority populations. e. 100 years and a redistribution of House of Representatives seats according to socioeconomic status.

The correct answer is C. The Constitution provides for a national census every 10 years, and a redistribution of House seats according to population shifts. Under the original Constitutional provision, the number of representatives was to be no more than one for every 30,000 citizens. There were 65 members in the first House, and the number was increased to 106 after the first census. Had the 1-to-30,000 formula been adhered to permanently, population growth in the United States would have brought the total number of representatives to about 7,000. Instead, the formula has been adjusted over the years, and today the ratio of representatives to people is about 1-to-600,000.

According to the Constitution, the electors in the Electoral College shall be appointed a. by the Congress. b. by random drawing in a lottery. c. in a manner to be determined by the state legislatures. d. by voters in an election. e. by the House of Representatives.

The correct answer is C. The Constitution provides for the establishment of the Electoral College, but it leaves to the state legislatures the question of how its electors are to be appointed. Subsequently, every state passed legislation providing for electors to be directly elected by the voters.

The European political philosophers whose writings influenced the concepts of liberty and government contained in the Declaration of Independence and the Constitution belonged to the intellectual and cultural movement known as a. the Reformation. b. the Renaissance. c. the Enlightenment. d. Realpolitik. e. the Great Awakening.

The correct answer is C. The Enlightenment was an eighteenth-century movement with roots as far back as the thirteenth-century revival of Aristotelian logic by St. Thomas Aquinas. The French political philosophers Jean-Jacques Rousseau and (Charles-Louis Secondat, Baron of) Montesquieu, for example, had a major influence on Thomas Jefferson and James Madison in their work on the Declaration of Independence and the Constitution, respectively.

The duties of the Federal Reserve Bank, established by Act of Congress in 1913, include all of the following EXCEPT a. conducting the nation's monetary policy. b. supervising and regulating banking institutions, and protecting the credit rights of consumers. c. enforcing tax judgments made by the Internal Revenue Service. d. maintaining the stability of the financial system. e. providing various financial services to the U.S. government, the public, financial institutions, and foreign official institutions.

The correct answer is C. The Federal Reserve Bank was founded in 1913 to provide the nation with a safer, more flexible, and more stable monetary and financial system. As America's central bank, the Fed fulfills many important duties. In conducting the nation's monetary policy, the Fed buys securities, which expands bank reserves, enabling banks to offer more loans and stimulate economic activity. When the Fed sells, it contracts bank reserves, reducing lending and slowing the economy. The Fed can also affect the economy by changing the interest rate at which domestic or foreign banks borrow money from the Federal Reserve System E—or by changing statutory rules regarding bank holdings. These actions are taken in the interests of maintaining the stability of the financial system and promoting a healthy economy. The Fed also oversees and regulates the nation's banking system, and it guards the credit rights of consumers.

Who were the authors of the Federalist Papers? a. John Adams, James Madison, and Thomas Jefferson b. Samuel Adams, John Jay, and John Quincy Adams c. Alexander Hamilton, James Madison, and John Jay d. James Madison, Thomas Jefferson and Andrew Jackson e. Alexander Hamilton, James Madison and Thomas Jefferson

The correct answer is C. The Federalist Papers—authored by Hamilton, Madison, and Jay—provided arguments for a stronger federal government.

If a bill passed by Congress remains with the President for more than ten days without him signing it, and Congress a. is no longer in session, the bill becomes law. b. is still in session, the bill is considered vetoed. c. is no longer in session, the bill has been "pocket vetoed." d. is still in session, the President has until the next session of Congress to sign the bill into law. e. is in session or is not in session, the bill becomes law.

The correct answer is C. The President has 10 days to sign legislation. However, if he fails to sign, the legislation will still become law. There is one exception to this rule. If the 10-day period extends past the adjournment of Congress, failure to sign will have the same effect as a veto. This presidential tactic is known as a "pocket veto." A bill that has been "pocket vetoed" is dead and, unlike other bills, cannot be reintroduced to Congress for a possible override.

Which of the following is true of a Presidential primary? a. The result determines who is President. b. There can be two winners. c. Only registered party members can vote. d. It is held in only key states. e. It is overseen by the Supreme Court.

The correct answer is C. The Presidential primary is the system of selecting a party's candidate for office in an intraparty election in which only registered members of that party may vote.

What explains the relatively high likelihood that the Supreme Court will overturn a decision if it chooses to hear an appeal? a. The Supreme Court judges cases by a more exacting standard than the lower courts. b. The Supreme Court tries to motivate Congress to enact new legislation by striking down outdated laws. c. The Supreme Court places the highest priority on hearing cases where it is clear that errors have been made. d. Since Supreme Court members are non-elected, they are able to oppose popular legislation and policy. e. The Supreme Court is more divided than circuit courts or state courts.

The correct answer is C. The Supreme Court has a very heavy caseload. The Justices must prioritize cases; therefore, they tend to hear those cases in which they feel errors have been made.

A temporary panel that works to reconcile different final House and Senate versions of a bill is known as a. a joint committee. b. a quorum. c. a conference committee. d. an executive session. e. a pro-forma session.

The correct answer is C. When there are differing Senate and House versions of legislation, a conference committee will attempt to create a compromise bill. Their final report must be adopted by a majority vote in both bodies before it can be sent to the President to be signed into law or sent back to Congress with a veto.

All of the following statements about the Voting Rights Act of 1965 are true EXCEPT: a. It directed the Attorney General to challenge the constitutionality of poll taxes. b. It suspended the use of literacy tests used to discriminate against poor and minority voters. c. It expressly prohibited the practice of gerrymandering. d. It required states in the covered jurisdiction to attain "preclearance" from the Attorney General before making changes to voting standards, practice, or procedure. e. It was amended in 1985 to prohibit vote dilution without requiring the proof of discriminatory purpose demanded by the original Act.

The correct answer is C. The Voting Rights Act of 1965 was designed to combat the systematic abridgement or denial of voting rights of minority voters, which had been ongoing for decades, especially in the South. While it prohibited discrimination against voters on the basis of race or color (a practice which led to vote dilution in these communities), it did not expressly outlaw gerrymandering. However, many lawsuits were raised in the aftermath of the Voting Rights Act, which called the controversial practice into further question and led to court-ordered redistricting in some areas. The preclearance provision D was renewed for 25 years in 1985, at the same time as the standard for proof of discriminatory purpose was eliminated.

The drafters of the Constitution realized that certain functions, such as sanitation, education, and local transportation, could be best served by a. the federal government. b. the Senate. c. local jurisdictions. d. individuals. e. foreign countries.

The correct answer is C. The drafters of the Constitution made the national structure supreme, but they wisely recognized the need for a series of governments more directly in contact with the people and more keenly attuned to their needs. Thus, certain functions—such as defense, currency regulation, and foreign relations—could only be managed by a strong centralized government. But others—such as sanitation, education, and local transportation—could be better served by local jurisdictions.

All of the following statements about the office of the Speaker of the House of Representatives are true EXCEPT: a. The Speaker is the highest authority in the House of Representatives. b. The Speaker has the ability to control action on the House floor and to steer the legislative agenda of the majority. c. The Speaker's office is responsible for determining which bills will be considered on the House floor, and when they will be considered. d. If a catastrophe were to claim the lives of both the President and the Vice-President, the Speaker would become President. e. On formal occasions, the Speaker represents the House in a ceremonial role.

The correct answer is C. The duty of producing the schedules and information that tells other representatives which bills are going to be considered, and when, falls to the office of the Majority Whip. With his ability to control action on the House floor and steer the legislative agenda, the Speaker is the highest authority in the House of Representatives. He also represents the House at official ceremonies. The Speaker of the House is third in line for the Presidency, after the President and the Vice-President.

The First Amendment's "free exercise clause" protects the rights of citizens to freely a. exercise their right to vote. b. express their views. c. practice their religion. d. manage their business. e. petition their leaders.

The correct answer is C. The free exercise clause guarantees that the government will not create laws that "prohibit the free exercise" of religion. This has been interpreted as barring laws that discourage or prevent individuals from practicing their religion.

Which of the following statements about Miranda v. Arizona is correct? a. It declared segregation by race unconstitutional. b. It granted women the same voting rights as men. c. It declared the recitation of the Pledge of the Union unconstitutional. d. It required that citizens be informed of their rights when being placed under arrest. e. It declared that a woman's reproductive rights were included in the conception of the right to privacy.

The correct answer is C. The only statement that can be reasonably inferred is the one that puts net interest payments (13%) higher than Medicaid spending (9%).

All of the following statements about the United States Department of Treasury are true EXCEPT: a. It oversees the United States Mint and the Bureau of Engraving and Printing. b. It oversees the Savings Bond Marketing Office, part of the Bureau of Public Debt. c. It is responsible for allocating funds to the various governmental departments and agencies. d. It guards against counterfeiting and other types of monetary fraud. e. It advises and assists in areas of domestic finance, banking, and other related economic matters.

The correct answer is C. The responsibility of allocating funds to the various governmental departments and agencies belongs to the Congress, specifically the Appropriations committees. The office of the Treasurer was founded September 6, 1777, twelve years before the establishment of the Department of the Treasury. It underwent a reorganization in 1981. Since that time, the Treasurer has overseen the U.S. Mint and the Bureau of Printing and Engraving as well as the Savings Bond Marketing Office.

In the recent past, government efforts to restrict free political expression have been justified mainly on the grounds of a. religion. b. public decency. c. national security. d. economics. e. accuracy.

The correct answer is C. The right of free political expression is central to the functioning of our democracy. Therefore, the government is unlikely to place restrictions on it unless it can be argued to threaten "national security." The government has not attempted to restrict free political expression for any of the other reasons given.

The Supreme Court hears a. all cases under its original jurisdiction. b. all cases under its appellate jurisdiction. c. most cases under its appellate jurisdiction and some cases under its original jurisdiction. d. most cases under its original jurisdiction and some cases under its appellate jurisdiction. e. a similar number of cases under its original and its appellate jurisdiction.

The correct answer is C. The vast majority of cases before the Supreme Court are appellate cases. However, the Constitution does grant the Court original jurisdiction in a few instances (i.e., in disputes between states). As a result, the Court hears mostly appellate cases and a small number of original jurisdiction cases.

The second highest level of the federal judiciary is made up of the a. Supreme Court. b. district courts. c. courts of appeals. d. district bankruptcy courts. e. city courts.

The correct answer is C. Under the Supreme Court, the second highest level of the federal judiciary is made up of the courts of appeals. The courts of appeals review decisions of the district courts (trial courts with federal jurisdiction) within their areas. They also are empowered to review orders of the independent regulatory agencies in cases where the internal review mechanisms of the agencies have been exhausted and there still exists substantial disagreement over legal points.

In 1872, Susan B Anthony was arrested in Rochester, New York for a. writing herself in as a candidate for mayor, because women were not permitted to run for office. b. leading a protest against unfair working conditions in a shirt factory. c. giving a speech that was deemed by local authorities to be un-American. d. casting a vote, because it was illegal for women to vote. e. tax evasion.

The correct answer is D. After meeting with Elizabeth Cady Stanton and other female suffragists at Seneca Falls, NY in 1848, Susan B. Anthony was arrested in Rochester, NY in 1872 for voting. Ultimately, she was fined $100, which she refused to pay. Wyoming territory was the first part of the U.S. to give women the right to vote in 1869. After years of political action by women's suffrage groups, the Nineteenth Amendment, giving women the right to vote, passed and was ratified in 1920.

Which of the following is NOT a political influence of the American media? a. Helping establish the agenda for political discussion by choosing which issues it reports on b. Determining which political candidates the public views as viable contenders for office c. Assessing the performance of elected officials and changing the public's perceptions of their leaders d. Changing election results through exit polls e. Ruining political campaigns through negative coverage of a candidate

The correct answer is D. Although polling can lower support for failing campaigns, "exit polling" is not an example of this. In an exit poll, voters leaving the polls are asked which candidate they chose. There is no proof of this activity having influenced an election. Each of the other answer choices represents a clear political influence of the American media.

The ranking minority party member of a Senate committee will a. preside over the committee in the chairperson's absence. b. determine which bills are heard before the committee. c. preside jointly with the ranking majority party member of the committee. d. become the committee chairperson if his or her party gains a majority in the Senate. e. be considered for the post of President Pro Tempore.

The correct answer is D. Committee chairs in the Senate are given to the majority party member with the greatest committee seniority. Since the ranking minority members will become the most senior majority members if their party wins a Senate majority, they can expect to become the committee chairperson.

The process of determining how many representatives a district will elect to the House of Representatives, based on U.S. Census population figures, is referred to as a. proctoring. b. redistricting. c. redistributing. d. apportionment. e. gerrymandering.

The correct answer is D. Congressional apportionment is the process by which the number of representatives to be elected by a given district is determined, using population data from the U.S. Census. Redistricting (B) is the process by which the borders of election districts are sometimes redrawn. Gerrymandering (E) refers to redistricting that is done in order to include a coveted group of voters within a certain district, or to avoid splitting a voting bloc.

Proponents of limited government, unregulated free markets, national self-reliance, and conventional social values are best describes as a. liberals. b. socialists. c. libertarians. d. conservatives. e. leftists.

The correct answer is D. Conservatives tend to favor less government involvement in the economy, strong support for the military, and government support for traditional moral values.

Which two related issues were most fiercely debated at the Constitutional Convention of 1787? a. Church and state b. Taxation and spending c. Liberty and security d. Representation and slavery e. Currency and banking

The correct answer is D. Delegates to the Constitutional Convention were divided over how to structure representation under the new Constitution. The Great Compromise that settled these differences satisfied both populous slave-owning southern states and less populous northern states. Larger states were happy that the new union would have a two-chamber legislature and, for the purpose of apportionment in the House of Representatives, slaves would count as three-fifths of a citizen. Smaller states were also satisfied that despite their small size, they would be represented by two senators.

Which of the following in NOT a due process right? a. The right to be free from unlawful search and seizure b. The right to retain an attorney c. The right to remain silent d. The right to trial by jury in all criminal and civil cases e. The right to not be tried twice for the same crime

The correct answer is D. Due process rights protect the accused and define the process the government must follow before punishing criminals. Although trial by jury is a due process right for criminal matters, the same does not apply to all civil cases.

An extended debate that is intended to indefinitely delay or prevent a vote in the Senate is known as a a. firewall. b. cloture. c. junket. d. filibuster. e. hold.

The correct answer is D. Filibusters are typically used as a last resort by the minority party to prevent passage of unwanted legislation. In the Senate, there is no limit to debate unless 60 votes are available to establish "cloture," ending the filibuster by limiting debate to three days.

Which of the following statements about Gideon v. Wainwright is correct? a. It established a cause of action for unconstitutional acts by federal agents. b. It declared that the government cannot criminalize anti-American speech. c. It declared that Congress has no power to enact a statute that limits the jurisdiction of federal courts. d. It declared that a state's refusal to appoint counsel for a defendant violated the "due process" clause of the Fourteenth Amendment. e. It declared the Constitution to be the "Supreme Law of the Land."

The correct answer is D. Gideon v. Wainwright was a 1963 case concerning a man, arrested for breaking and entering, whose request for a court-appointed lawyer was denied in accordance with Florida state law, which provided indigent defendants with counsel only in capital cases. Writing for the majority, Justice Hugo L. Black argued that Florida's forcing the defendant to serve as his own lawyer violated the Fourteenth Amendment, which guarantees that a citizen may not be deprived of life, liberty, or property without undergoing "due process of law."

In what instance will the House of Representatives decide the winner of a presidential election? a. When the candidate with the most electoral votes fails to win the popular vote b. When no candidate wins a majority of the popular vote c. When a candidate wins the electoral vote but doesn't win a majority of the states d. When no candidate wins the majority of the votes in the Electoral College e. When a disputed election causes uncertainty over the actual electoral vote totals

The correct answer is D. If no candidate wins a majority of the votes in the Electoral College, the House of Representatives must decide which of the three candidates with the most electoral votes will become President. A disputed election will result in court cases that will then naturally be decided by the legislative branch.

Which of the following describe challenges a member of the cabinet typically faces during his or her first year in office? I. Trying to avoid being manipulated by the bureaucracy he/she has been appointed to oversee. II. Dealing with entrenched lobbying organizations that may not have the public interest as their primary concern. III. Vying for the Supreme Court's attention, particularly in a time of war or crisis. a. I only b. II only c. III only d. I and II only e. I, II, and III

The correct answer is D. One of the toughest difficulties a cabinet member may face is trying to avoid being manipulated by the very bureaucracy he or she has been appointed to head (I). Interest groups constantly lobby the bureaucracy (II), as well as the legislature in an attempt to influence legislation. However, the cabinet has little direct association with the Supreme Court and would rarely compete for its attention (III).

The Constitution does NOT give Congress the authority to a. impeach and remove federal judges. b. refuse to ratify treaties negotiated by the President. c. override a presidential veto. d. pardon people convicted of federal crimes. e. raise taxes and authorize spending.

The correct answer is D. Only the President has the authority to pardon citizens who have been convicted of federal crimes.

Which of the following is NOT an example of congressional oversight? a. Committee inquiries and hearings b. Senate advice and consent for presidential nominations c. House of Representatives impeachment proceedings and Senate trials d. Override of a presidential veto e. Formal consultations with and reports from the President

The correct answer is D. Overriding a presidential veto is a power of Congress but not a power of oversight. Congressional oversight allows the legislature to keeps tabs on Executive performance, prevent waste and fraud, safeguard civil rights, and collect information crucial to the law-making process.

The stories that the media chooses to cover a. reveal the conservative bias of the media. b. undermine the efforts made by leadership in more important areas. c. attest to the liberal bias of the media. d. can help determine the country's political agenda. e. have little effect on politics.

The correct answer is D. Perhaps the media's greatest power is its ability to help determine the nation's political agenda by the stories it chooses to cover. This power is often not centralized or organized, and it may merely reflect the tastes and opinions of the voters themselves.

Which of the following most influenced the founders of the Constitution to establish a federal republic in America? a. Political theories that federalism best protected individual liberties b. Examples of successful federal governments in Europe c. The need to reconcile slave-owning and free states in one system d. State governments already existed and refused to dissolve themselves e. A desire to limit popular participation in national decision making

The correct answer is D. Practical considerations accounted for the founders' decision to create a federal republic. While the states were determined to keep as much power as possible, they recognized the need for a unified government to deal with internal and external challenges.

Since 1980, there has been proposed legislation to abolish all of the following federal departments or agencies EXCEPT a. Department of Housing and Urban Development. b. Department of Energy. c. Department of Education. d. Federal Communications Commission. e. National Endowment for the Arts.

The correct answer is D. President Ronald Reagan promised during his 1980 campaign that if elected, he would seek to abolish the Department of Education. In 1996, the Republican platform included a plank to abolish the Department of Housing and Urban Development. A bill proposing the abolishment of the Department of Energy was introduced in the Senate in 1996, but failed to pass. Bills proposing abolishment of the National Endowment for the Arts were introduced by Senate Republicans in 1997 and 2002 but failed to pass.

Which of the following is NOT a right of due process? a. Right to a fair and public trial conducted in a competent manner b. Taxes may only be taken for public purposes c. Right to an impartial jury d. Private property cannot be taken for public use e. Right to be present at the trial

The correct answer is D. Private property can be taken for public use under eminent domain. However, due process does require that the owners of taken property must be fairly compensated.

All of the following are reason why incumbent representatives usually win re-election to Congress EXCEPT: a. Incumbents use franking privileges to communicate with voters. b. Incumbents generate good will by getting "pork barrel" projects for their districts. c. Incumbents have large staffs to work on constituency relations. d. Incumbents always benefit from congressional redistricting. e. Incumbents receive the most money from special interest groups.

The correct answer is D. Representatives often lose support if their party is in control of their home state's legislature when redistricting occurs.

Strict constructionists interpret the Constitution a. inflexibly and disapprove of judicial restraint. b. broadly and attempt to rule in the spirit of the Constitution. c. historically and attempt to correct the shortcomings of the Constitution. d. literally and grant only those freedoms expressly cited in the Constitution. e. contextually and extend freedoms to all citizens.

The correct answer is D. Strict constructionists believe that the Constitution should not be "interpreted." Instead, they believe that judges should rule based on the exact text of the Constitution as written by the framers. For example, a strict constructionist would oppose a woman's right to reproductive freedom based on the argument that a "right to privacy" (as defined in Roe v. Wade) is not written into the Constitution.

Studies show that the effect of advance polls on voter behavior was more pronounced in the 1990s than in previous decades. This was probably the result of a. the use of polling to predict primary as well as general elections. b. computer advances in polling data collection and tabulation. c. several highly publicized elections in which polls failed to predict the outcome accurately. d. the growth of the Internet and rise of cable and satellite television networks as a medium for the dissemination of poll results. e. the use of the Internet to gather polling data.

The correct answer is D. Studies show the rising impact of advance polls can be ascribed to the increased influence of Internet and cable television news sources. In modern political campaigns, a politician's "approval rating" could almost be likened to his score in an athletic contest—in other words, it has become nearly all-important.

All of the following statements about the New Deal legislation passed during the Great Depression under the administration of President Franklin D. Roosevelt are true EXCEPT: a. It resulted in the creation of many new federal agencies and established a permanent "welfare state." b. It expanded presidential power. c. It opened the American power structure to new groups and helped bring about the political realignment of the mid-1930s. d. It took America in an increasingly isolationist foreign policy direction. e. It established a system of social rights to replace dependence on private charity.

The correct answer is D. The "New Deal" refers to the sweeping, anti-depression legislation passed during President Franklin D. Roosevelt's first two terms in office. To its supporters, the New Deal embodied laudable humanitarian goals, an openness to new ideas, and a willingness to expand federal powers to achieve its ends (B). Its critics saw it as an egregious expansion of the bureaucracy that failed to end the Great Depression, while creating a burdensome "welfare state" in the process. The broad New Deal legislation had many far-reaching consequences. It established Social Security and other social-rights programs, to replace dependence on private charities, and reformed the relationships between labor and management, and government and finance. However, it did not take America in an isolationist foreign policy direction.

Which of the following Supreme Court cases refuted the doctrine of "separate but equal"? a. Baker v. Carr b. McCulloch v. Maryland c. Garcia v. San Antonio d. Brown v. Board of Education e. Miranda v. Arizona

The correct answer is D. The 1954 case of Brown v. Board of Education overturned the 1896 ruling of Plessy v. Ferguson, which had approved the standard of "separate but equal." In Brown v. Board of Education, the Supreme Court decided that separate schooling could never be equal and ordered that all schools be desegregated.

The Bill of Rights contains amendments related to all of the following areas EXCEPT a. freedom of speech. b. the right to bear arms. c. the right to a speedy trial. d. the right to sexual freedom. e. freedom not to have soldiers stationed in one's home

The correct answer is D. The Bill of Rights does not address sexual freedom. A is Amendment 1; B is Amendment 2; C is Amendment 6; E is Amendment 3.

The Department of Labor, founded in 1888, was established initially as a bureau of which federal department? a. Agriculture b. Commerce c. War d. Interior e. State

The correct answer is D. The Bureau of Labor was established in 1884 as a division of the Department of the Interior. The Department of the Interior started as the Home Department in 1849. In 1872, Yellowstone, the first national park, was established. In 1873, Congress transferred territorial oversight to the Department of the Interior from the Department of State. The DOI created the U.S. Geological Survey in 1879.

Which of the following are roles of the Central Intelligence Agency? I. Aid local police forces to combat white-collar crime II. Coordinating the nation's intelligence activities III. Evaluating and disseminating intelligence that affects national security a. I only b. II only c. III only d. II and III only e. I, II, and III

The correct answer is D. The Central Intelligence Agency was created in 1947 with the signing of the National Security Act by President Truman. The National Security Act charged the Director of Central Intelligence (DCI) with coordinating the nation's intelligence activities and correlating, evaluating and disseminating intelligence which affects national security. Statement I is a role of the FBI.

The presidential action that frees an individual of criminal charges is a a. reprieve. b. commutation. c. parole. d. pardon. e. probation.

The correct answer is D. The Constitution grants the President the ability to pardon any person convicted of crimes. Presidents normally pardon a number of individuals just before the end of their tenure.

The Constitution grants Congress the power to do all of the following EXCEPT a. borrow money for the public treasury. b. declare war. c. govern commerce among the states and with foreign countries. d. tax exports from any one state. e. coin money and set its value.

The correct answer is D. The Constitution prohibits Congress from treating one state differently than the others. Taxing the exports of one state but not the others would be unfair and unconstitutional.

The following were original positions in President George Washington's cabinet in 1789 EXCEPT a. Secretary of State. b. Secretary of War. c. Secretary of the Treasury. d. Secretary of Commerce. e. Attorney General.

The correct answer is D. The Department of Commerce and Labor was established by act of Congress in February, 1903. The other four positions made up the entirety of the first presidential cabinet.

Which Supreme Court case revolutionized libel law in the United States by deciding that public officials could not sue successfully for libel simply by proving that published information is false? a. Gideon v. Wainwright b. Miranda v. Arizona c. National Labor Relations Board v. Jones & Laughlin Steel Corp. d. New York Times Co. v. Sullivan e. Dickerson v. United States

The correct answer is D. The First Amendment to the U.S. Constitution guarantees freedom of the press, but for years the Supreme Court refused to use the First Amendment to protect the media from libel lawsuits—lawsuits based on the publication of false information that damages a person's reputation. The Supreme Court's ruling in New York Times Co. v. Sullivan revolutionized libel law in the United States by deciding that public officials could not sue successfully for libel simply by proving that published information is false. The Court ruled that the complainant also must prove that reporters or editors acted with "actual malice" and published information "with reckless disregard of whether it was false or not."

Which of the following principles, contained in the Fourth Amendment, protects a citizen from unwarranted search and seizure? a. Due process b. Habeus corpus c. Checks and balances d. Probable cause e. Exclusionary rule

The correct answer is D. The Fourth Amendment to the Constitution provides aggressive protection against unwarranted search and seizure. Law-enforcement authorities must demonstrate probable cause before a court will issue a search warrant for that location. The principle of due process A protects citizens against imprisonment without proper trial, while a writ of habeus corpus (B) is a judicial mandate to a prison official ordering that an inmate be brought before the court, usually in order to determine whether or not the imprisonment is lawful. The exclusionary rule (E) states that evidence obtained by police in an illegal search and seizure operation can be excluded from a trial.

The current number of representatives in the Senate is a. 2. b. 25. c. 50. d. 100. e. 435.

The correct answer is D. The Senate has two representatives from each of the 50 states.

Which of the following is an example of how an interest group may have an influence on the federal judiciary? a. The American Medical Association may seek limits on jury awards in malpractice cases. b. The American Civil Liberties Union may bring suit to improve the rights of prisoners. c. The National Association for the Advancement of Colored Peoples may question the constitutionality of laws that affect its constituents. d. The American Bar Association may be asked to evaluate potential candidates for the federal court. e. The National Rifle Association may protest greater restrictions on the sale and purchase of assault rifles.

The correct answer is D. The judiciary often asks the American Bar Association to evaluate candidates for the federal court. Otherwise, the judiciary is as isolated as possible from the influences of interest groups. Such influences may be part of the democratic process for Congress but would be seen as unethical for the judiciary.

The largest entitlement program of the federal government is a. Food Stamps. b. Medicaid. c. National Defense. d. Social Security. e. Medicare

The correct answer is D. The largest entitlement program is Social Security. (Medicare and Medicaid are also large but not as large as Social Security.) Entitlement programs deliver aid to all individuals who meet the eligibility conditions specified by law.

The most effective way for an interest group to form a powerful policy coalition of interests is to ally with a. the Majority Whip and state governors. b. the Secretary of State and a congressional subcommittee. c. a federal agency and the Attorney General. d. a federal agency and a congressional subcommittee. e. a political action committee and a major television news network.

The correct answer is D. The most effective way for an interest group to form a powerful policy coalition of interests is to ally with a federal agency and a congressional subcommittee. By working hand-in-hand with the bureaucracy and a congressional subcommittee, an interest group greatly improves its chances of influencing new legislation in the Congress.

Which of the following is not a standing committee of the United States Congress? a. Agriculture b. Appropriations c. Veterans' Affairs d. Investigations e. Budget

The correct answer is D. There is no committee for investigations, although Congress may create committees to conduct investigations if the need arises. All of the other answer choices are standing (permanent) committees of both the House and Senate.

Does the President have any control over the sessions of Congress? a. No, under no circumstances can the President call a session of Congress. b. No, but the Vice President can call a session of Congress if necessary. c. Yes, but the President can only call a session of the Senate and not the House of Representatives. d. Yes, the President can call a session of Congress "on extraordinary occasions." e. Yes, the President can call a session of Congress whenever he wants.

The correct answer is D. Under the Constitution, the President may convene Congress, or either House, "on extraordinary occasions." It is usual for the President in calling an extra session to indicate the exact matter that needs the attention of Congress. However, once convened, a Congress cannot be limited in the subject matter that it will consider.

Which of the following statements regarding labor unions is true? a. Unions may require their members to contribute money to PACs. b. Unions historically have endorsed more Republican candidates than Democratic ones. c. Unions may disobey a court injunction to return to work during a strike. d. Many Americans belong to unions, making endorsement by unions a coveted political prize. e. Union members who are employed by the federal government may not engage in strikes.

The correct answer is D. Unions are prohibited from demanding political contributions as a condition of membership. Historically, organized labor has sided more often with Democrats than with Republicans. Unions have the right to strike, but they do not have the right to ignore a court injunction ordering them back to work.

The increased concentration of mass media does which of the following? I. Narrows the range of political perspectives available to most Americans II. Lowers journalistic standards for the reporting of political issues III. Grants media owners a large amount of political influence a. I only b. II only c. III only d. I and III only e. I, II, and III

The correct answer is D. While some criticism has been made of the major networks for focusing on human interest stories rather than political news, there is little indication that concentration lowers journalistic standards. Rather, concentration threatens the diversity of views presented and may grant a dangerous amount of influence to those who have control over a large sector of the American mass media.

Which of the following has NOT been linked to major realignments among American voters? a. Disunity at the party convention and disagreements over key policies b. Involvement of a third party in prior elections c. Unusually high voter turnout d. Intensive television advertising campaigns e. Real social and economic problems

The correct answer is D. With the exception of intensive advertising, all of the listed factors have resulted in party realignments among American voters. Although advertising can change the outcome of individual elections, it has not yet caused voters to change party en masse to vote for the opposition.

The Constitution requires that state governments, like the federal government, must be a. legal in form, with the final authority resting with the state legislators. b. federal in form, with final authority resting with the elected officials. c. republican in form, with the final authority resting with the governor. d. democratic in form, with final authority resting with the people. e. unified in form, with the final authority resting with the state constitution.

The correct answer is D. Within the limits of the Constitution, each state must recognize and respect the laws of the others. State governments, like the federal government, must be democratic in form, with final authority resting with the people.

Members of Congress are expected to a. become experts in all areas of policy so that they can be informed whenever an important vote arises. b. defer important votes to their constituents so that the people's voice can be heard. c. be absent from Congressional sessions if previously scheduled events conflict. d. consult with the President if they are unsure about how to vote on a particular bill. e. specialize in a few policy areas rather than claim expertise in the whole range of legislative concerns.

The correct answer is E. Congress is composed of committees, and the committee members are expected to specialize in the subject of that committee. Representatives who are not members of a particular committee read the committee reports and listen to presentations so that they can understand the various issues at hand.

A budget resolution is a a. joint resolution between the United States and foreign allies to impose boycotts on rogue nations. b. resolution put forth by the President that must be approved by state legislatures before it goes into effect. c. resolution adopted by the House of Representatives, but not the Senate, that must be approved by the Cabinet before it is passed into law. d. resolution adopted by the Senate, but not the House or Representatives, that must be approved by the President before it is passed into law. e. concurrent resolution, adopted by both Houses of Congress, that sets forth a Congressional budget plan for the budget year and at least four out-years.

The correct answer is E. A budget resolution is a concurrent resolution, adopted by both Houses of Congress, that sets forth a Congressional budget plan for the budget year and at least four out-years. The plan consists of spending and revenue targets with which subsequent appropriation acts and authorization acts that affect revenues and direct spending are expected to comply. The targets established in the budget resolution are enforced in each House of Congress through procedural mechanisms set out in law and the rules of each House.

In American politics, the term "caucus" refers to a. a preliminary vote usually taken early in the electoral process within a party to express a preference for one of the party's candidates. b. the group of electors who meet on election day and formally select the next President of the United States. c. a formal statement of position on major political issues drafted by a candidate or a political party. d. a political party outside the two-party system that is perceived to have a significant base of support. e. the main mechanism used by modern political parties to nominate their candidate for President.

The correct answer is E. A caucus is the main mechanism used by modern political parties to nominate their candidate for President. In the Presidential nomination process, it now denotes a meeting of local party activists at the precinct level who select, in an open forum, delegates to county meetings. These delegates in turn select delegates to state meetings, and these state-level conventions select delegates to the party's national convention.

In addition to overseeing the activities of one or more federal agencies, a congressional oversight committee may also a. investigate the activities of individual members of Congress, recommending disciplinary action if necessary. b. amend the rules with regard to procedure in the Senate or the House of Representatives. c. oversee the ratification of an international treaty. d. oversee the ratification of an amendment to the Constitution. e. serve as the authorizing committee for federal agencies' programs and operations.

The correct answer is E. A congressional oversight committee is charged with overseeing the activities of one or more federal agencies. It is common for the oversight committee also to serve as the authorizing committee for the federal agencies' programs and operations.

A decision of the Supreme Court a. can never be overturned. b. can be overturned by the President. c. must be a unanimous decision by all the justices. d. must be approved by Congress. e. cannot be appealed to any other court.

The correct answer is E. A decision by the Supreme Court cannot be appealed to any other court. It can, however, be overturned by the Supreme Court if a case is returned to the Court on appeal, ruling out A, B and D are false because they mix the branches of government, which the Constitution explicitly separates. C is also wrong; Supreme Court decisions are based on the majority opinion and are rarely unanimous.

Which of the following tactics were used to deny African Americans the right to vote? I. Poll taxes II. Literacy tests III. Grandfather clauses a. I only b. II only c. III only d. I and III only e. I, II, and III

The correct answer is E. All of these methods were used to prevent African Americans from exercising their right to vote. Similar tests were often not applied to any class of white voters.

The fact that few Presidents have been able to command the legislative process for a significant length of time gives evidence that a. the legislative process is more influenced by the judicial branch than by the executive branch. b. the President is often busy with his other responsibilities. c. legislators' voting patterns may change depending on public opinion polls. d. Congress enjoys taking over and controlling legislation originally suggested by the President. e. there is an inherent weakness built into the office of the Presidency.

The correct answer is E. All three branches of the government have their inherent strengths and weakness, according to the doctrine of separation of powers, which guards American democracy against despotic rule. In his public speeches and private dealings with Congress, the President will often propose legislation and attempt to set the tone for the legislative session. However, separation of powers ensures that the President can never assume complete control over the decisions of Congress.

Why must the House of Representatives readopt rules of procedure at the beginning of each new session, while the Senate has not done so since 1789? a. Unlike the House of Representatives, the Senate gives the individual states an equal say in rules as in legislation. b. The House of Representatives committee structure aggravates political infighting and leads to frequent rule changes. c. Although the Constitution outlines Senate procedure, it gives House of Representatives members the freedom to create their own rules. d. While a two-thirds majority is needed to amend Senate rules, only a simple majority is needed to change those of the House of Representatives. e. The Senate is a continuing body, while House of Representatives members must stand for election every two years.

The correct answer is E. At least two-thirds of the previous Senate will remain after each election while all Representatives must defend their seats every two years. While the rules of the Senate have changed over time, this has been done through amendments to existing rules rather than the wholesale adoption of a new set of rules. Over time, House rules have changed a great deal more than those of the Senate.

Both the Senate and the House of Representatives have a standing committee on a. Indian Affairs. b. Women's Issues. c. Space Exploration. d. Religious Oversight. e. Armed Services.

The correct answer is E. Both houses have a committee for armed forces. A belongs to the Senate. No committees exist for B, C, or D.

In presidential campaigns, federal matching funds are provided to a. all candidates. b. minor party candidates only. c. major party candidates only. d. winning candidates only. e. candidates who pass the fundraising threshold.

The correct answer is E. Candidates are eligible for federal matching funds after they receive at least $5,000 in contributions from individuals in twenty different states. There are no restrictions by party.

All of the following statements regarding the Articles of Confederation of 1781 are true EXCEPT: a. They created a national government that was largely devoid of power, while retaining most of the power in the states. b. They were intended to pacify those who feared the power of a remote national government. c. They set forth the country's name, "The United States of America." d. They allowed for the extradition of fugitives from one state to another for the purpose of facing criminal trial. e. They established a bicameral legislature, with each state having two votes.

The correct answer is E. The Articles of Confederation established a unicameral Congress with each state having one vote. The Articles proved insufficient, establishing a precariously weak national government, while concentrating too much power in the states. This was done largely to pacify those who had suffered under British rule and who feared similar abuses by the new national government. In addition to declaring the country's name and the right of the federal government to coin money, levy taxes, and send diplomats to foreign lands, the Articles set guidelines for cooperation among the states in matters of national defense and law enforcement. Article IV allowed for the extradition of fugitives from any state back to the state in which they had been charged or convicted.

Which of the following are "checks" granted to Congress by the Constitution? I. The ability to decide the number of Supreme Court justices II. The ability to decide the appellate jurisdiction of the Supreme Court III. The right to impeach and remove federal judges a. I only b. II only c. III only d. I and II only e. I, II, and III

The correct answer is E. The Congress has substantial powers to check the judiciary including the three listed above. However, Congress has rarely tried to use these powers.

The number of electoral votes that a state possesses is determined by the a. state's population. b. number representatives that the state has in the House of Representatives. c. number of voters registered with the major parties. d. population at the time of joining the union. e. number Representatives and Senators that the state has in Congress.

The correct answer is E. The Constitution establishes that each state should have a number of electors equal to its number of representatives in the House plus two for its two senators. Since the number of House representatives is based on state population, larger states have more electors. There are currently 538 electors, and 270 are needed to win the Presidency.

The Constitution grants justices of the Supreme Court terms of a. four years. b. six years. c. ten years. d. twelve years. e. life tenure.

The correct answer is E. The Constitution grants justices life tenures. However, the Constitution granted Congress a check of judicial power by granting the power to impeach Supreme Court justices.

All of the following are found in the Declaration of Independence EXCEPT a. an assertion of the basic right of the people to abolish and replace an unjust government with a just one. b. an indictment of King George III for repeated usurpations of the colonists' rights. c. a formal declaration of the independence of the thirteen "united colonies" from Britain. d. an explanation of problems caused by British troop presence in the colonies. e. a defense of the Articles of Confederation.

The correct answer is E. The Declaration of Independence declared the thirteen colonies' independence from Britain. However, it did not establish a political system. After stating the right of the people to abolish and replace an unjust government, the Declaration makes a strong case against King George, citing, among other things, the hated practice of quartering British troops in colonists' homes. The Articles of Confederation were drafted in 1777 and finally ratified by the thirteen colonies in 1781.

The Bill of Rights incorporates protections for all of the following EXCEPT a. right to bear arms. b. freedom to petition the government. c. freedom of religion. d. equality before the law. e. pursuit of happiness.

The correct answer is E. The Declaration of Independence, not the Bill of Rights, identifies the basic human rights of "life, liberty, and the pursuit of happiness."

Which Executive department was created first? a. Commerce b. Defense c. Labor d. Transportation e. State

The correct answer is E. The Department of State was created in 1789. The Departments of Commerce, Labor, and Transportation were all created in the twentieth century. In 1947, the Department of Defense was made by combining the Departments of War, Navy, and Air Force.

The mission of the Department of the Interior is to a. ensure that the military is well prepared for any challenge it faces. b. protect the nation's children from abuse and unfair labor laws. c. uphold the moral and religious authority that guides the nation. d. maintain relations and distribute information between the states. e. protect and provide access to the nation's natural and cultural heritage.

The correct answer is E. The Department of the Interior protects America's treasures for future generations, provides access to our nation's natural and cultural heritage, offers recreation opportunities, honors responsibilities to American Indians and Alaska Natives, conducts scientific research, provides wise stewardship of energy and mineral resources, fosters sound use of land and water resources, and conserves and protects fish and wildlife.

Which of the following are responsibilities of the Council of Economic Advisers? I. To assist and advise the President in the preparation of the Economic Report II. To gather timely and authoritative information concerning economic developments and trends III. To advise the President on the extent to which various programs and activities of the federal government are contributing or not contributing to the overall economic policy IV. To develop national economic policies that promote free enterprise and maintain employment, production, and purchasing power a. I only b. IV only c. I and III d. II and IV e. I, II, III, and IV

The correct answer is E. The Employment Act of 1946 established the Council of Economic Advisers. As the President's chief advisers on the economy, the CEA is charged with making studies and providing the President with the best available information on the economy (I, II). The CEA also advises the President on whether or not various government programs have been successful at fulfilling their stated objectives (III), and develops economic policies designed to maintain the economy's strength and minimize the effects of fluctuations (IV).

Which of the following is NOT considered a civil liberty? a. Freedom of speech b. The right of equal protection c. The right of due process d. Freedom of religion e. The right to work

The correct answer is E. There is no "right to work" included in the Constitution; it is not a civil liberty.

Which of the following best describes the Supreme Court's current ruling on prayer in American public schools? a. Any form of speech, including prayer, is protected under the First Amendment no matter where it takes place. b. Scheduled voluntary prayer is acceptable because children are not coerced into religious behavior. c. Prayer is acceptable at all times so long as it is "student-initiated." d. Prayer in schools may take place during a school mandated "moment of silence." e. Mandated prayer in schools violates the "Establishment Clause" of the First Amendment.

The correct answer is E. The First Amendment's "Establishment Clause" prohibits the government from using its power to "establish" a religion. Prayer in schools and the numerous strategies for getting around "prayer" have all been rejected by the Supreme Court as violation of the essentially secular nature of American government.

The agency that provides Congress with evaluations of public policies is the a. Department of Justice. b. Department of Commerce. c. Office of Personnel Management. d. Office of Management and Budget. e. General Accounting Office.

The correct answer is E. The General Accounting Office is the audit, evaluation, and investigative arm of Congress. It exists to evaluate the effectiveness of public policies for Congress.

Which of the following is NOT a government corporation? a. U.S. Postal Service b. Import / Export Bank of the United States c. Amtrak d. Tennessee Valley Authority e. National Science Foundation

The correct answer is E. The National Science Foundation is an independent agency of the federal government, but it is not a government corporation. Government corporations are at least partially self-funding, semi-autonomous agencies with clearly defined responsibilities.

Which of the following is a duty of the President of the United States? a. Overseeing special congressional committees b. Determining interest rates c. Drafting state budgets for federal programs d. Impeaching Supreme Court Justices e. Appointing representatives to foreign countries

The correct answer is E. The President is responsible for appointing ambassadors to other nations across the world, including an ambassador to the United Nations.

The Reconstruction Finance Corporation (RFC), founded in 1932 by President Herbert Hoover to help combat the Great Depression, was the predecessor of which one of the following federal agencies? a. Securities and Exchange Commission (SEC) b. Federal Deposit Insurance Corporation (FDIC) c. Department of Housing and Urban Development (HUD) d. Freddie Mac e. Small Business Administration (SBA)

The correct answer is E. The Reconstruction Finance Corporation (RFC) was the base of operations of a federal lending program for all businesses, large and small, that had been hurt by the Great Depression. In 1942, to help engage America's small businesses in the war effort, the Smaller War Plants Corporation (SWPC) was formed. There was a movement to abolish the RFC in the early 1950s. Prior to the creation of the SBA by President Dwight D. Eisenhower in 1953, the Office of Small Business (OSB) of the Department of Commerce provided government support for small businesses. The SBA today offers low interest loans and other programs designed to help small businesses increase their competitiveness.

The major agency of fiscal policy is the a. Council of Economic Advisors. b. Office of Management and Budget. c. Export-Import Bank. d. Federal Reserve Board. e. Treasury Department.

The correct answer is E. The Treasury Department is responsible for the management of the federal debt and the printing of currency. It is the major fiscal policy agency. The Federal Reserve Board is responsible for monetary policy.

The Ways and Means Committee of the House of Representatives has jurisdiction over which of the following? I. Raising the revenue required to finance the federal government by levying taxes II. Supervising the authority of the federal government to borrow money III. Overseeing Social Security and other social insurance programs IV. Shielding American companies from unfair competition by levying tariffs on foreign goods a. I only b. III only c. I, III, and IV only d. I, II, and III only e. I, II, III, and IV

The correct answer is E. The Ways and Means Committee of the House of Representatives is responsible for raising the revenue necessary for the operation of the federal government (I), supervising the national debt (II), overseeing Social Security and other benefits programs (III), and imposing tariffs and passing legislation to protect and stimulate trade (IV).

All of the following were important platforms of the nineteenth-century Republican party EXCEPT a. opposition to the spread of slavery. b. preservation of the Union. c. abolition of slavery. d. post-Civil War reconstruction. e. government pension for all Civil War veterans.

The correct answer is E. The early Republican party was known for its diversity of interests and personalities. Among the earliest planks in the party platform, in the years leading up to the Civil War, were opposition to the spread of slavery (A) and preservation of the Union (B). Led by President Abraham Lincoln, the party toughened its stance on slavery, ultimately calling for its abolition (C). Following the Civil War, the party advocated the Reconstruction of the South (D), as well as basic civil rights for the former slaves. The Republicans advocated a pension for Union veterans only.

Which of the following methods may be used to propose an amendment to the United States Constitution? I. By the passage of a two-thirds vote in both houses of Congress II. By the request of three-fourths of the state legislatures III. By a national constitutional convention called by Congress on the request of two-thirds of the state legislatures a. I only b. II only c. III only d. I and II only e. I and III only

The correct answer is E. The methods for proposal and ratification of Constitutional amendments differ. While support from three-fourths of the state legislatures can ratify amendments to the Constitution, this method cannot be used to propose Constitutional amendments. Amendments can only be proposed through the steps outlined in statements I and III.

All of the following are principles upon which the United States Constitution was founded EXCEPT: a. Every state is equal and the national government cannot give special rights to an individual state. b. There should be three branches of government to make, execute, and interpret the nation's laws. c. All men are equal before the law regardless of social standing. d. The best government is a government by laws not by men. e. All citizens should have the right to keep and bear arms.

The correct answer is E. The right to bear arms is the Second Amendment in the Bill of Rights; it was not a founding principle of the U.S. Constitution. All of the other answers deal with general principles of American governmental philosophy and structure rather than particular rights.

One reason why the United States has only two major political parties is that a. the federal government does not allow third-party representatives to attend Congress. b. third parties are not bound by the same campaign laws as are Democrats and Republicans. c. the Constitution limits the number of political parties that the nation can have at any one time. d. almost everyone in the United States has the same beliefs and moral systems. e. one or both of the major parties often adopt a third party's most popular issues, thereby taking its voters.

The correct answer is E. Third parties have a hard time surviving because one or both of the major parties often adopt their most popular issues, and thus their voters. The most successful third party in recent years has been Ross Perot's Reform Party, which had some success in the Presidential elections of 1992 and 1996. Jesse Ventura became the first Reform Party candidate to win statewide office when he was elected governor of Minnesota in 1998.

Under the Articles of Confederation, most power was concentrated in the a. federal government. b. judiciary. c. legislature. d. executive. e. states.

The correct answer is E. Under the Articles of Confederation, the states maintained most governmental powers, ceding to the national government only the power to direct foreign policy and conclude treaties.

Which of the following is a mandatory qualification of United States Senators? a. Must have donated money to the states from which they are elected b. Must have attended school in the states from which they are elected c. Must have been born in the states from which they are elected d. Must have parents who live in the states from which they are elected e. Must be residents of the states from which they are elected

The correct answer is E. United States Senators must be residents of the states from which they are elected. Other qualifications are that they must be at least 30 years of age and citizens of the United States for at least nine years.

All of the following profiles fit the typical Republican voter EXCEPT that they are a. men. b. college graduates. c. wealthy individuals. d. Christians. e. urban dwellers.Powers unambiguously granted to the Congress by the Constitution are known as a. enumerated powers. b. implied powers. c. resulting powers. d. inherent powers. e. delegated powers.

The correct answer is E. Urban Americans are more likely to be Democrats. All of the other categories are associated with higher support for the Republican Party.

When partisanship increases in national institutions, which of the following may result? I. Voting patterns in Congress may shift, affecting the lawmaking process. II. The President may alter his tactics for setting the agenda for Congress, in response to partisan trends. III. There may be increased competition between the President and his party to define their public identities. a. I only b. II only c. I and II only d. I and III only e. I, II, and III

The correct answer is E. When partisanship increases in national institutions, it means that parties themselves, and not simply their leaders, are gaining in power and influence. A partisan trend may cause votes in Congress to split along party lines more predictably than before the trend occurred (I). A partisan shift in voting patterns in Congress may require the President to change his tactics when setting the agenda for Congress (II). When the President's own party gains power and influence, it more puts pressure on the President to conform to the will of the party, leading to increased competition between the President and the party to define their public identities (III).

Which of the following committee assignments would confer the most power and influence on members of the Senate? a. Budget b. Energy and Natural Resources c. Environment and Public Works d. Governmental Affairs e. Appropriations

The correct answer is E. While Budget A; Energy and Natural Resources B; Environment and Public Works C; and Governmental Affairs (D) are all important committees, none of them ranks as the most powerful or influential. Appropriations (E) is the most powerful committee in the Senate because it decides exactly how and where tax monies will be spent; therefore, it confers the most power and influence on its members.


Related study sets

Cards, Cars, and Currency Lesson 2 Econ Lowdown

View Set

CIS 156: Python Programming - Strings (Chapter 7)

View Set

[Music Theory] chapter 07: triads

View Set

chapter 7 (middle childhood: body and mind)

View Set

Upper limb trapezius, levator scapulae, and rhomboid major + minor pg 300

View Set

T3: Lesson 2 - The Origins of Hinduism & Buddhism -- Text 2: The Caste System Shapes India

View Set

Adobe Photoshop CC18 Practice Training Test

View Set